Two days after a myocardial infarction, a client begins reporting orthopnea and dyspnea

Try the new Google Books

Check out the new look and enjoy easier access to your favorite features

Two days after a myocardial infarction, a client begins reporting orthopnea and dyspnea


Page 2

Charles Wiener 

Two days after a myocardial infarction, a client begins reporting orthopnea and dyspnea
 Cynthia D. Brown 
Two days after a myocardial infarction, a client begins reporting orthopnea and dyspnea
 Anna R. Hemnes

DIRECTIONS: Choose the one best response to each question.

1. A 62-year-old man presents to his physician complaining of shortness of breath. All of the following findings are consistent with left ventricular dysfunction as a cause of the patient’s dyspnea EXCEPT:

A. Feeling of chest tightness

B. Nocturnal dyspnea

C. Orthopnea

D. Pulsus paradoxus greater than 10 mmHg

E. Sensation of air hunger

2. A 48-year-old man is evaluated for hypoxia of unknown etiology. He recently has noticed shortness of breath that is worse with exertion and in the upright position. It is relieved with lying down. On physical examination, he is visibly dyspneic with minimal exertion. He is noted to have a resting oxygen saturation of 89% on room air. When lying down, his oxygen saturation increases to 93%. His pulmonary examination shows no wheezes or crackles. His cardiac examination findings are normal without murmur. His chest radiograph reports a possible 1-cm lung nodule in the right lower lobe. On 100% oxygen and in the upright position, the patient has an oxygen saturation of 90%. What is the most likely cause of the patient’s hypoxia?

A. Circulatory hypoxia

B. Hypoventilation

C. Intracardiac right-to-left shunting

D. Intrapulmonary right-to-left shunting

E. Ventilation–perfusion mismatch

3. A patient is evaluated in the emergency department for peripheral cyanosis. All of the following are potential etiologies EXCEPT:

A. Cold exposure

B. Deep venous thrombosis

C. Methemoglobinemia

D. Peripheral vascular disease

E. Raynaud’s phenomenon

4. A 35-year-old woman is seen in clinic for evaluation of dyspnea. Which of the following physical findings would fit the diagnosis of idiopathic pulmonary arterial hypertension?

A. Elevated neck veins, normal S1 and S2, II/VI diastolic blowing murmur heard at the right upper sternal border

B. Elevated neck veins; singular, loud S2; II/VI systolic murmur left lower sternal border

C. Elevated neck veins; loud, fixed, split S2; III/VI systolic murmur left lower sternal border

D. Elevated neck veins, expiratory splitting of S2, II/VI harsh systolic murmur left upper sternal border

E. Elevated neck veins, barrel chest, prolonged expiratory phase

5. A 75-year-old woman with widely metastatic non–small cell lung cancer is admitted to the intensive care unit with a systolic blood pressure of 73/25 mmHg. She presented complaining of fatigue and worsening dyspnea over the last 3–5 days. Her physical examination shows elevated neck veins. Chest radiograph shows a massive, water bottle–shaped heart shadow and no new pulmonary infiltrates. Which of the following additional findings is most likely present on physical examination?

A. Fall in systolic blood pressure greater than 10 mmHg with inspiration

B. Lack of fall of the jugular venous pressure with inspiration

C. Late diastolic murmur with opening snap

D. Pulsus parvus et tardus

E. Slow y-descent of jugular venous pressure tracing

6. A 78-year-old man is admitted to the intensive care unit with decompensated heart failure. He has long-standing ischemic cardiomyopathy. Electrocardiogram (ECG) shows atrial fibrillation and left bundle branch block. Chest radiograph shows cardiomegaly and bilateral alveolar infiltrates with Kerley’s B-lines. Which of the following is least likely to be present on physical examination?

A. Fourth heart sound

B. Irregular heart rate

C. Pulsus alternans

D. Reversed splitting of the second heart sound

E. Third heart sound

7. A 45-year-old man is admitted to the intensive care unit with symptoms of congestive heart failure. He is addicted to heroin and cocaine and uses both drugs daily via injection. His blood cultures have yielded methicillin-sensitive Staphylococcus aureus in four of four bottles within 12 hours. His vital signs show a blood pressure of 110/40 mmHg and a heart rate of 132 beats/min. There is a IV/VI diastolic murmur heard along the left sternal border. A schematic representation of the carotid pulsation is shown in Figure 7. What is the most likely cause of the patient’s murmur?

Two days after a myocardial infarction, a client begins reporting orthopnea and dyspnea

FIGURE 7

A. Aortic regurgitation

B. Aortic stenosis

C. Mitral stenosis

D. Mitral regurgitation

E. Tricuspid regurgitation

8. A 72-year-old man seeks evaluation for leg pain with ambulation. He describes the pain as an aching to crampy pain in the muscles of his thighs. The pain subsides within minutes of resting. On rare occasions, he has noted numbness of his right foot at rest, and pain in his right leg has woken him at night. He has a history of hypertension and cerebrovascular disease. Four years previously, he had a transient ischemic attack and underwent right carotid endarterectomy. He currently takes aspirin, irbesartan, hydrochlorothiazide, and atenolol on a daily basis. On examination, he is noted to have diminished dorsalis pedis and posterior tibial pulses bilaterally. The right dorsal pedis pulse is faint. There is loss of hair in the distal extremities. Capillary refill is approximately 5 seconds in the right foot and 3 seconds in the left foot. Which of the following findings would be suggestive of critical ischemia of the right foot?

A. Ankle-brachial index less than 0.3

B. Ankle-brachial index less than 0.9

C. Ankle-brachial index greater than 1.2

D. Lack of palpable dorsalis pedis pulse

E. Presence of pitting edema of the extremities

9. A 24-year-old man is referred to cardiology after an episode of syncope while playing basketball. He has no recollection of the event, but he was told that he collapsed while running. He awakened lying on the ground and suffered multiple contusions as a result of the fall. He has always been an active individual but recently has developed some chest pain with exertion that has caused him to restrict his activity. His father died at age 44 while rock climbing. He believes his father’s cause of death was sudden cardiac death and recalls being told his father had an enlarged heart. On examination, the patient has a III/VI mid-systolic crescendo-decrescendo murmur. His electrocardiogram shows evidence of left ventricular hypertrophy. You suspect hypertrophic cardiomyopathy as the cause of the patient’s heart disease. Which of the following maneuvers would be expected to cause an increase in the loudness of the murmur?

A. Hand-grip exercise

B. Squatting

C. Standing

D. Valsalva maneuver

E. A and B

F. C and D

10. An 18-year-old college freshman is being evaluated for a heart murmur heard at health screening. She reports an active lifestyle, no past medical history, and no cardiac symptoms. She has a mid-systolic murmur that follows a nonejection sound and crescendos with S2. The murmur duration is greater when going from supine to standing and decreases when squatting. The murmur is heard best along the lower left sternal border and apex. Her electrocardiogram is normal. Which of the following is the most likely condition causing the murmur?

A. Aortic stenosis

B. Hypertrophic obstructive cardiomyopathy

C. Mitral valve prolapse

D. Pulmonic stenosis

E. Tricuspid regurgitation

11. Which of the following characteristics makes a heart murmur more likely to be caused by tricuspid regurgitation than mitral regurgitation?

A. Decreased intensity with amyl nitrate

B. Inaudible A2 at the apex

C. Prominent c-v wave in jugular pulse

D. Onset signaled by a mid-systolic click

E. Wide splitting of S2

12. You are examining a 25-year-old patient in clinic who came in for a routine examination. Cardiac auscultation reveals a second heart sound that is split and does not vary with respiration. There is also a grade 2–3 mid-systolic murmur at the midsternal border. Which of the following is most likely?

A. Atrial septal defect

B. Hypertrophic obstructive cardiomyopathy

C. Left bundle branch block

D. Normal physiology

E. Pulmonary hypertension

13. Left bundle branch block is indicative of which of the following sets of conditions?

A. Atrial septal defect, coronary heart disease, aortic valve disease

B. Coronary heart disease, aortic valve disease, hypertensive heart disease

C. Coronary heart disease, aortic valve disease, pulmonary hypertension

D. Pulmonary embolism, cardiomyopathy, hypertensive heart disease

E. Pulmonary hypertension, pulmonary embolism, mitral stenosis

14. A 57-year-old man with long-standing ischemic cardiomyopathy is seen in the clinic for a routine visit. He reports good compliance with his diuretic regimen, but has seen his weight fall about 2 kg since his last visit. Routine chemistries are drawn and show a potassium value of 2.0 meq/L. The patient is referred to the emergency department for repletion of potassium. Which of the following is likely to be found on ECG before administration of potassium?

A. Diminution of P wave amplitude

B. Osborne waves

C. Prolongation of QT interval

D. Prominent U waves

E. Scooped ST segments

15. A 55-year-old woman from El Salvador is seen in the emergency department because of gradual onset of dyspnea on exertion. She denies chest pain, cough, wheezing, sputum, or fever. Her chest radiograph is notable for large pulmonary arteries and left atrial enlargement, but no parenchymal infiltrate. ECG shows a tall R in lead V1 and right axis deviation. Which of the following is most likely to be found on her echocardiography?

A. Aortic regurgitation

B. Aortic stenosis

C. Low left ventricular ejection fraction

D. Mitral stenosis

E. Tricuspid stenosis

16. A 29-year-old woman is in the intensive care unit with rhabdomyolysis due to compartment syndrome of the lower extremities after a car accident. Her clinical course has been complicated by acute renal failure and severe pain. She has undergone fasciotomies and is admitted to the intensive care unit. An ECG is obtained (shown in Figure 16). What is the most appropriate course of action at this point?

Two days after a myocardial infarction, a client begins reporting orthopnea and dyspnea

FIGURE 16

A. 18-lead ECG

B. Coronary catheterization

C. Hemodialysis

D. Intravenous fluids and a loop diuretic

E. Ventilation/perfusion imaging

17. Acute hyperkalemia is associated with which of the following electrocardiographic changes?

A. Decrease in the PR interval

B. Prolongation of the ST segment

C. Prominent U waves

D. QRS widening

E. T-wave flattening

18. The ECG shown below (Figure 18) was most likely obtained from which of the following patients?

A. A 33-year-old female with acute-onset severe headache, disorientation, and intraventricular blood on head CT scan

B. A 42-year-old male with sudden-onset chest pain while playing tennis

C. A 54-year-old female with a long history of smoking and 2 days of increasing shortness of breath and wheezing

D. A 64-year-old female with end-stage renal insufficiency who missed dialysis for the last 4 days

E. A 78-year-old male with syncope, delayed carotid upstrokes, and a harsh systolic murmur in the right second intercostal space

Two days after a myocardial infarction, a client begins reporting orthopnea and dyspnea

FIGURE 18

19. You are evaluating a new patient in your clinic who has brought in the ECG shown below (Figure 19) to the visit. The ECG was performed on the patient 2 weeks ago. What complaint do you expect to elicit from the patient?

Two days after a myocardial infarction, a client begins reporting orthopnea and dyspnea

FIGURE 19

A. Angina

B. Hemoptysis

C. Paroxysmal nocturnal dyspnea

D. Pleuritic chest pain

E. Tachypalpitations

20. All the following ECG findings are suggestive of left ventricular hypertrophy EXCEPT:

A. (S in V1 + R in V5 or V6) greater than 35 mm

B. R in aVL greater than 11 mm

C. R in aVF greater than 20 mm

D. (R in I + S in III) greater than 25 mm

E. R in aVR greater than 8 mm

21. Based on the electrocardiogram below (Figure 21), treating which condition might specifically improve this patient’s tachycardia?

Two days after a myocardial infarction, a client begins reporting orthopnea and dyspnea

FIGURE 21

A. Anemia

B. Chronic obstructive pulmonary disease (COPD)

C. Myocardial ischemia

D. Pain

22. A 75-year-old man is undergoing routine cardiac catheterization for evaluation of stable angina that has not responded to medical therapy. He is inquiring about the risks associated with the procedure. Which of the following is the most common complication of cardiac catheterization and coronary angiography?

A. Acute renal failure

B. Bradyarrhythmias

C. Myocardial infarction

D. Tachyarrhythmias

E. Vascular access site bleeding

23. Which of the following patients is an appropriate candidate for right heart catheterization?

A. A 54-year-old woman with dyspnea of unclear etiology; a loud, fixed split second heart sound; normal chest radiograph; and evidence of bidirectional shunt across her interatrial septum

B. A 54-year-old man with an episode of sustained monomorphic ventricular tachycardia while at the casino terminated with bystander defibrillation. After arrival in the emergency department, the patient is hemodynamically stable.

C. A 63-year-old woman with a history of tobacco abuse, hypercholesterolemia, and type 2 diabetes mellitus with chest pain at rest, a normal ECG, and mild elevation in serum troponin value

D. A 66-year-old man with a history of diabetes and hypercholesterolemia brought to the emergency department with 1 hour of substernal chest pain and shortness of breath. His blood pressure is 95/60 mmHg with a heart rate of 115 beats/min. An ECG shows a new left bundle branch block since his prior ECG 1 month ago.

E. A 79-year-old man seen in the cardiology clinic for evaluation of severe aortic stenosis found on echocardiography performed for evaluation of dyspnea

24. A 55-year-old woman is undergoing evaluation of dyspnea on exertion. She has a history of hypertension since age 32 and is also obese with a body mass index (BMI) of 44 kg/m2. Her pulmonary function tests show mild restrictive lung disease. An echo-cardiogram shows a thickened left-ventricular wall, left-ventricular ejection fraction of 70%, and findings suggestive of pulmonary hypertension with an estimated right-ventricular systolic pressure of 55 mmHg, but the echocardiogram is technically difficult and of poor quality. She undergoes a right heart catheterization that shows the following results:

Two days after a myocardial infarction, a client begins reporting orthopnea and dyspnea

What is the most likely cause of the patient’s dyspnea?

A. Chronic thromboembolic disease

B. Diastolic heart failure

C. Obstructive sleep apnea

D. Pulmonary arterial hypertension

E. Systolic heart failure

25. Which of the following is a risk factor for the development of thromboembolism in patients with the tachycardia-bradycardia variant of sick sinus syndrome?

A. Age greater than 50 years

B. Atrial enlargement

C. Diabetes mellitus

D. Prothrombin 20210 mutation

E. None of the above; there is no increased risk of thromboembolism with the tachycardia-bradycardia variant of sick sinus syndrome.

26. A 38-year-old man is evaluated for the recent onset of feeling fatigued. He is a busy executive and active triathlete. He competed a challenging course 1 week earlier without difficulty but feels tired at other times. Laboratory examination, including hematocrit and TSH, are unremarkable. Because his wife reports occasional snoring, a sleep study is recommended. There are no notable apneas, but ECG monitoring during the night shows sinus bradycardia. His heart rate varies between 42 and 56 while sleeping. His resting heart rate while awake is 65–72 beats/min. Which of the following is the most appropriate management for his bradycardia?

A. Carotid sinus massage

B. Intermittent nocturnal wakening

C. Measurement of free T4

D. No specific therapy

E. Referral for pacemaker placement

27. All of the following are reversible causes of sinoatrial node dysfunction EXCEPT:

A. Hypothermia

B. Hypothyroidism

C. Increased intracranial pressure

D. Lithium toxicity

E. Radiation therapy

28. A 58-year-old man is admitted to the hospital after experiencing 2 days of severe dyspnea. Three weeks ago he had an ST elevation myocardial infarction that was treated with thrombolytics. He reports excellent adherence to his medical regimen that includes atorvastatin, lisinopril, metoprolol, and aspirin. On examination, his heart rate is 44 beats/min, his blood pressure is 100/45 mmHg, his lungs have bilateral crackles, and his cardiac examination is notable for elevated neck veins, bradycardia, and 2+ bilateral leg edema. There are no gallops or new murmurs. ECG shows sinus bradycardia and evidence of the recent infarct, but no acute changes. Which of the following is the most appropriate next management step?

A. Begin dopamine

B. Hold metoprolol

C. Measure TSH

D. Refer for pacemaker placement

E. Refer for urgent coronary angiography

29. A 23-year-old college student home for the summer is evaluated in the emergency department for dizziness that began within the last 3 days. He reports a rash on his right leg that looked like a target several days ago, but is otherwise healthy. Physical examination shows bradycardia at 40 beats/min and blood pressure of 88/42 mmHg; oxygen saturation is normal. His examination is otherwise unremarkable except for a bulls-eye rash over the right upper thigh. ECG shows third-degree AV block. Which of the following laboratory studies is most likely to reveal the etiology of his signs and symptoms?

A. ANA

B. HLA B27 testing

C. Borrelia burgdorferi ELISA

D. RPR

E. SCL-70

30. In the tracing below (Figure 30), what type of conduction abnormality is present and where in the conduction pathway is the block usually found?

Two days after a myocardial infarction, a client begins reporting orthopnea and dyspnea

FIGURE 30

A. First-degree AV block; intranodal

B. Second-degree AV block type 1; intranodal

C. Second-degree AV block type 2; infranodal

D. Second-degree AV block type 2; intranodal

31. A 47-year-old woman with a history of tobacco abuse and ulcerative colitis is evaluated for intermittent palpitations. She reports that for the last 6 months every 2–4 days she notes a sensation of her heart “flipflopping” in her chest for approximately 5 minutes. She has not noted any precipitating factors and has not felt lightheaded or had chest pains with these episodes. Her physical examination is normal. A resting ECG reveals sinus rhythm and no abnormalities. Aside from checking serum electrolytes, which of the following is the most appropriate testing?

A. Abdominal CT with oral and IV contrast

B. Event monitor

C. Holter monitor

D. Reassurance with no further testing needed

E. Referral for EP study

32. After further testing, the patient in question 31 is found to have several episodes of atrial premature contractions. Which of the following statements regarding the dysrhythmia in this patient is true?

A. Atrial premature contractions are less common than ventricular premature contractions on extended ECG monitoring.

B. Echocardiography is indicated to determine if structural heart disease is present.

C. Metoprolol should be initiated for symptom control.

D. The patient should be reassured that this is not a dangerous condition and does not require further evaluation.

E. The patient should undergo a stress test to determine if ischemia is present.

33. A 55-year-old man with end-stage COPD is admitted to the intensive care unit with an exacerbation of his obstructive lung disease. Because of hypercarbic respiratory failure, he is intubated and placed on assist-control mechanical ventilation. Despite aggressive sedation, his ventilator alarms several times that peak inspiratory pressures are high. The physician is called to the bedside to evaluate tachycardia. Examination is notable for a blood pressure of 112/68 mmHg and heart rate of 180 beats/min. Cardiac examination shows a regular rhythm, but no other abnormality. Breath sounds are decreased on the right. ECG shows narrow complex tachycardia. With carotid sinus massage, the heart rate transiently drops to 130 beats/min, but then returns to 180 beats/min. Which of the following is the most appropriate next step in management?

A. Adenosine 25-mg IV push

B. Amiodarone 200-mg IV push

C. Chest radiograph

D. Metoprolol 5-mg IV push

E. Sedation followed by cardioversion

34. All of the following are risk factors for stroke in a patient with atrial fibrillation EXCEPT:

A. Diabetes mellitus

B. History of congestive heart failure

C. History of stroke

D. Hypertension

E. Left atrial size greater than 4.0 cm

35. Which of the following statements regarding restoration of sinus rhythm after atrial fibrillation is true?

A. Dofetilide may be safely started on an outpatient basis.

B. In patients who are treated with pharmacotherapy and are found to be in sinus rhythm, a prolonged Holter monitor should be worn to determine if anticoagulation could be safely stopped.

C. Patients who have pharmacologically maintained sinus rhythm after atrial fibrillation have improved survival compared with patients who are treated with rate control and anticoagulation.

D. Recurrence of atrial fibrillation is uncommon when pharmacotherapy is used to maintain sinus rhythm.

36. A 57-year-old woman with a history of a surgically corrected atrial septal defect in childhood presents to the emergency department with palpitations for 3 days. She is found to have a heart rate of 153 beats/min and blood pressure of 128/75 mmHg, and an ECG shows atrial flutter. An echocardiogram demonstrates moderate right and left atrial dilation, postoperative changes from her surgery, and normal left and right ventricular function. Which of the following is true?

A. Anticoagulation with dabigatran should be initiated.

B. If a transesophageal echocardiogram does not demonstrate left atrial thrombus, she may be cardioverted without anticoagulation.

C. Intravenous heparin should be started immediately.

D. She should be immediately cardioverted.

E. Transthoracic echocardiogram is adequate to rule out the presence of left atrial thrombus.

37. A patient presents with palpitations and shortness of breath for 6 hours. In the emergency department waiting room an ECG is performed (shown in Figure 37). Which of the following is most likely to be found on physical examination?

Two days after a myocardial infarction, a client begins reporting orthopnea and dyspnea

FIGURE 37

A. Diffuse abdominal tenderness with guarding

B. Diffuse expiratory polyphonic wheezing with poor air movement and hyperinflation

C. Left ventricular heave and third heart sound

D. Supraclavicular lymphadenopathy

E. Vesicular rash over right T5 dermatome

38. A 43-year-old woman is seen in the emergency department after sudden onset of palpitations 30 minutes prior to her visit. She was seated at her work computer when the symptoms began. Aside from low back pain, she is otherwise healthy. In triage, her heart rate is 178 beats/min, and blood pressure is 98/56 mmHg with normal oxygen saturation. On physical examination, she has a “frog sign” in her neck and tachycardia, but is otherwise normal. ECG shows a narrow complex tachycardia without identifiable P waves. Which of the following is the most appropriate first step to manage her tachycardia?

A. 5 mg metoprolol IV

B. 6 mg adenosine IV

C. 10 mg verapamil IV

D. Carotid sinus massage

E. DC cardioversion using 100 J

39. A 37-year-old man who is healthy aside from a prior knee surgery is evaluated in the emergency department for palpitations that developed suddenly while eating dinner. He is found to have a heart rate of 193 beats/min, blood pressure of 92/52 mmHg, and normal oxygen saturation. His physical examination is normal aside from tachycardia and mild diaphoresis. An ECG obtained before his knee surgery shows delta waves in the early precordial leads. His current ECG shows wide complex tachycardia. Which of the following therapies is contraindicated for treatment of his tachyarrhythmia?

A. Adenosine

B. Carotid sinus massage

C. DC cardioversion

D. Digoxin

E. Metoprolol

40. In an ECG with wide complex tachycardia, which of the following clues most strongly supports the diagnosis of ventricular tachycardia?

A. Atrial-ventricular dissociation

B. Classic right bundle branch block pattern

C. Irregularly irregular rhythm with changing QRS complexes

D. QRS duration greater than 120 milliseconds

E. Slowing of rate with carotid sinus massage

41. A 40-year-old male with diabetes and schizophrenia is started on antibiotic therapy for chronic osteomyelitis in the hospital. His osteomyelitis has developed just under an ulcer where he has been injecting heroin. He is found suddenly unresponsive by the nursing staff. His electrocardiogram is shown in Figure 41. The most likely cause of this rhythm is which of the following substances?

Two days after a myocardial infarction, a client begins reporting orthopnea and dyspnea

FIGURE 41

A. Furosemide

B. Metronidazole

C. Droperidol

D. Metformin

E. Heroin

42. Normal sinus rhythm is restored with electrical cardioversion in the patient in question 41. A 12-lead electrocardiogram is notable for a prolonged QT interval. Besides stopping the offending drug, the most appropriate management for this rhythm disturbance should include intravenous administration of which of the following?

A. Amiodarone

B. Lidocaine

C. Magnesium

D. Metoprolol

E. Potassium

43. You are caring for a patient with heart rate–related angina. With minor elevations in heart rate, the patient has anginal symptoms that impact his quality of life. On review of a 24-hour Holter monitor, it appears that the patient has sinus tachycardia at the time of his symptoms. What is the mechanism for this patient’s arrhythmia?

A. Delayed afterdepolarizations

B. Early afterdepolarizations

C. Increased automaticity

D. Reentry pathway

44. Where are the most common drivers of atrial fibrillation anatomically located?

A. Left atrial appendage

B. Mitral annulus

C. Pulmonary vein orifice

D. Sinus venosus

E. Sinus node

45. Symptoms of atrial fibrillation vary dramatically from patient to patient. A patient with which of the following clinical conditions will likely be the most symptomatic (e.g., short of breath) if the patient develops atrial fibrillation?

A. Acute alcohol intoxication

B. Hypertrophic cardiomyopathy

C. Hyperthyroidism

D. Hypothermia

E. Postoperative after thoracotomy

46. A 47-year-old postmenopausal woman is seen for onset of severe dyspnea over the last few weeks. She reports no preceding chest pain, cough, sputum, or fever, though she does report leg swelling. Physical examination is notable for a blood pressure of 145/78 mmHg and heart rate of 123 beats/min. Exophthalmos is present as well as bilateral inspiratory crackles occupying approximately one-third of the lower chest; neck vein distention; normal cardiac rhythm, though tachycardia is present; and a third heart sound with no murmur. Bilateral lower extremity edema and a fine hand tremor are also present. Which of the following is the most likely pathophysiologic explanation for her heart failure?

A. Anemia with high-output state

B. Chronic systemic hypertension with resultant left ventricular hypertrophy and nonsystolic heart failure

C. Hemochromatosis with subsequent restrictive cardiomyopathy

D. Myocardial infarction with depressed left ventricular systolic function

E. Thyrotoxicosis with high-output state

47. Which of the following statements is true regarding measurement of plasma BNP to diagnose heart failure?

A. An elevated plasma BNP in a dyspneic patient confirms the diagnosis of left heart failure.

B. In the presence of renal failure, BNP levels are suppressed even when heart failure is present.

C. Plasma BNP levels may be falsely low in patients with obesity and heart failure.

D. Serial measurement of BNP in the therapy of decompensated heart failure should be used to guide therapy.

E. All of the above are true.

48. A 64-year-old man with an ischemic cardiomyopathy, ejection fraction 35%, and stage C heart failure is seen in the cardiology clinic for evaluation of his disease status. The patient reports a regular exercise regimen of walking on the treadmill several times weekly and occasional exacerbations of his leg edema that he manages with an extra dose of furosemide. He has never been hospitalized for heart failure. His current medical regimen includes lisinopril, aspirin, furosemide, atorvastatin, digoxin, spironolactone, and metoprolol. He is interested in stopping medications because of their expense. Which of the following statements is true regarding his medical regimen?

A. ACE inhibition therapy has not been shown to improve heart failure symptoms.

B. Beta blocker therapy in this patient may be exacerbating his occasional need for extra furosemide and therefore should be stopped.

C. He should be switched from spironolactone to eplerenone for improved efficacy, as seen in patients with EF less than 35%.

D. If digoxin is withdrawn, he will likely have worsening symptoms.

E. If he is intolerant to lisinopril because of cough, it would be reasonable to switch him to an angiotensin-receptor blocker.

49. A 78-year-old slender woman is seen in the emergency department after several weeks of dyspnea on exertion that progressed to dyspnea at rest following a summer cookout where she consumed multiple pickled vegetables. She also complains of leg swelling, orthopnea, and occasionally awakening at night with dyspnea. Her past medical history is notable for long-standing systemic hypertension, uterine prolapse, and an anxiety disorder. Examination confirms the presence of heart failure with a laterally displaced and sustained point of maximum impulse and a fourth heart sound. She is admitted to the hospital and given diuretics, and an echo-cardiogram is obtained. Echocardiography reveals severe left ventricular hypertrophy with an ejection fraction of 70%, but there are no focal wall motion abnormalities, and aortic and mitral valvular function is intact. Her right ventricular systolic pressure is estimated to be 45 mmHg. After resolution of her heart failure symptoms with diuresis, the patient is ready for discharge. Which of the following medications have been shown to improve mortality in patients with heart failure with preserved ejection fraction and should be included in this patient’s regimen?

A. Digoxin

B. Lisinopril

C. Metoprolol

D. Sildenafil

E. None of the above

50. A 68-year-old man with a history of myocardial infarction and congestive heart failure is comfortable at rest. However, when walking to his car he develops dyspnea, fatigue, and sometimes palpitations. He must rest for several minutes before these symptoms resolve. His New York Heart Association classification is which of the following?

A. Class I

B. Class II

C. Class III

D. Class IV

51. The husband of a 68-year-old woman with congestive heart failure is concerned because his wife appears to stop breathing for periods of time when she sleeps. He has noticed that she stops breathing for approximately 10 seconds and then follows this with a similar period of hyperventilation. This does not wake her from sleep. She does not snore. She feels well rested in the morning but is very dyspneic with even mild activity. What is your next step in management?

A. Electroencephalography

B. Maximize heart failure management

C. Nasal continuous positive airway pressure (CPAP) during sleep

D. Obtain a sleep study

E. Prescribe bronchodilators

52. A 53-year-old man undergoes cardiac transplantation for end-stage ischemic cardiomyopathy due to an underlying familial hypercholesterolemic disorder. His donor was a 23-year-old motor vehicle accident victim. The patient does well for the first 3 years after transplantation with only a single episode of acute rejection. He shows good compliance with his immunosuppression regimen, which includes prednisone and sirolimus. He is evaluated at a routine follow-up visit and reports that he has developed dyspnea on exertion. His pulmonary function tests are unchanged and a chest radiograph is normal. He undergoes right and left heart catheterization with biopsy of the transplanted heart. Severe, diffuse, concentric, and longitudinal coronary artery disease is found on coronary angiography, and histology shows no evidence of acute rejection. Which of the following statements is true regarding the coronary atherosclerosis found in this patient?

A. No immunosuppressive regimen has been shown to have a lower incidence of coronary atherosclerosis after cardiac transplantation.

B. The coronary atherosclerosis is most likely immunologic injury of the vascular endothelium in the transplanted organ.

C. The current coronary atherosclerosis after cardiac transplant is likely due to atherosclerosis present prior to transplantation.

D. The patient’s underlying cholesterol disorder did not predispose him to recurrent coronary atherosclerosis after cardiac transplantation.

E. Therapy with statins has not been associated with a reduced incidence of this complication of transplantation.

53. Which of the following is a known complication of ventricular assist device placement in patients with end-stage heart failure?

A. Cerebrovascular accident

B. Infection of insertion site

C. Mechanical device failure

D. Thromboembolism

E. All of the above

54. All of the following are potential complications of an atrial septal defect in adults EXCEPT:

A. Air embolism from a central venous catheter

B. Arterial oxygen desaturation with exertion

C. Embolic cerebrovascular accident

D. Pulmonary arterial hypertension

E. Unstable angina

55. A 32-year-old woman is seen by her primary care physician clinic for routine follow-up of her hypothyroidism. She also has a history of complex congenital heart disease with a partially corrected VSD with predominantly right to left shunt across her patch. She is doing well and is able to work in janitorial services without severe dyspnea. She denies any heart failure or neurologic symptoms, but does have a peripheral oxygen saturation of 78%. A routine CBC is drawn and shows a hematocrit of 65%. Which of the following is the most appropriate management of her elevated hematocrit?

A. Begin oxygen therapy

B. Check co-oximetry on arterial blood gas sample

C. Check serum erythropoietin level

D. Expectant waiting

E. Refer to hematology for phlebotomy

56. A 43-year-old man recently was found to have an asymptomatic atrial septal defect that was closed using a percutaneous patch 1 month ago without complication. He is undergoing a root canal at the dentist next week and calls his primary care office to determine if antibiotic prophylaxis is indicated. Which of the following statements is true regarding antibiotic prophylaxis in this patient?

A. Because he had only simple congenital heart disease, no prophylaxis is indicated.

B. Because the lesion is corrected, no prophylaxis is indicated.

C. He should avoid potentially bacteremic dental procedures unless no other alternative is available.

D. Routine antibiotic prophylaxis is indicated for bacteremic dental procedures, particularly if the patch is less than 6 months old.

E. Routine antibiotic prophylaxis is indicated for bacteremic dental procedures whenever foreign material is present.

57. A 20-year-old man undergoes a physical examination with chest radiograph for enrollment in the military. He has had a normal childhood without any major illness. There is no history of sinusitis, pneumonia, or chronic respiratory disease. Chest radiograph shows dextrocardia. On closer physical examination, a spleen tip is palpable on the right of the abdomen and the liver can be percussed on the left. Which of the following is true regarding his condition?

A. He is likely to have aortic stenosis.

B. He is likely to have aspermia.

C. He is likely to have an atrial septal defect.

D. He is likely to have a ventriculoseptal defect.

E. He is likely to otherwise be normal.

58. A 24-year-old male seeks medical attention for the recent onset of headaches. The headaches are described as “pounding” and occur during the day and night. He has had minimal relief with acetaminophen. Physical examination is notable for a blood pressure of 185/115 mmHg in the right arm, a heart rate of 70 beats/min, arterioventricular (AV) nicking on funduscopic examination, normal jugular veins and carotid arteries, a pressure-loaded PMI with an apical S4, no abdominal bruits, and reduced pulses in both lower extremities. Review of symptoms is positive only for leg fatigue with exertion. Additional measurement of blood pressure reveals the following:

Two days after a myocardial infarction, a client begins reporting orthopnea and dyspnea

Which of the following diagnostic studies is most likely to demonstrate the cause of the headaches?

A. MRI of the head

B. MRI of the kidney

C. MRI of the thorax

D. 24-hour urinary 5-HIAA

E. 24-hour urinary free cortisol

59. The patient described in question 58 is most likely to have which of the following associated cardiac abnormalities?

A. Bicuspid aortic valve

B. Mitral stenosis

C. Preexcitation syndrome

D. Right bundle branch block

E. Tricuspid atresia

60. Mitral stenosis is frequently complicated by pulmonary hypertension. Which of the following is a cause of pulmonary hypertension in mitral stenosis?

A. Interstitial edema in the walls of small pulmonary vessels

B. Passive transmission of elevated left atrial pressure

C. Obliterative changes in the pulmonary vascular bed

D. Pulmonary arteriolar constriction

E. All of the above

61. A 58-year-old man with a history of systemic hypertension, hyperlipidemia, and tobacco abuse is admitted to the intensive care unit with crushing chest pain associated with ST-segment elevation and small precordial Q waves. Because his symptoms have been present for 36 hours, he is not a candidate for thrombolytics. On admission to the ICU, his systemic blood pressure is 123/67 mmHg, heart rate is 67 beats/min after beta blockade, and his oxygenation saturation is 93% on 2L nasal cannula. The remainder of the physical examination is normal. He is treated with lisinopril, aspirin, heparin, and metoprolol. Before transfer can be arranged to a tertiary center, the patient reports extreme dyspnea. He is found to be diaphoretic and to have a heart rate of 80 beats/min, blood pressure of 84/56 mmHg, and oxygen saturation of 93% on 100% non-rebreather. His lungs have bilateral crackles throughout, and neck veins are moderately elevated. ECG is unchanged. Chest radiograph shows new alveolar infiltrates in the right lung greater than the left. Which of the following is a likely finding on physical examination?

A. A fourth heart sound, III/VI systolic murmur heard best at the apex with a “cooing” quality that radiates to the axilla

B. A right ventricular heave, loud second heart sound, III/VI murmur increasing with inspiration at the right lower sternal border

C. A third heart sound, III/VI crescendo-decrescendo murmur heard best at the right upper sternal border

D. Diffuse urticarial reaction, wheezing on pulmonary examination

E. Mucosal edema, finger swelling, stridor

62. Which of the following is the most appropriate next step in therapy for the patient in question 61?

A. Aerosolized albuterol

B. Initiation of norepinephrine infusion

C. Intravenous infusion of nitroprusside

D. Intravenous methylprednisolone

E. Placement of intraaortic balloon pump

63. A 26-year-old healthy woman is seen for a pap smear at a routine office visit. She feels well and has no complaints and no significant past medical history. Her internist performs a full physical examination and a mid-systolic click is heard. No murmur or gallop is present. She is concerned about this finding. Which of the following statements is true regarding her examination finding?

A. In most patients with this disorder, an underlying cause such as a heritable disorder of connective tissue is found.

B. Infective endocarditis prophylaxis is indicated for dental procedures potentially associated with bacteremia.

C. Most patients are asymptomatic from this lesion and will remain so their entire life.

D. She should begin therapy with aspirin 325 mg po daily.

E. This disorder cannot be visualized on echocardiography.

64. A 78-year-old man is evaluated for the onset of dyspnea on exertion. He has a long history of tobacco abuse, obesity, and diabetes mellitus. His current medications include metformin, aspirin, and occasional ibuprofen. On physical examination his peripheral pulses show a delayed peak and he has a prominent left ventricular heave. He is in a regular rhythm with a IV/VI mid-systolic murmur that is loudest at the base of the heart and radiates to the carotid arteries. A fourth heart sound is present. Echocardiography confirms severe aortic stenosis without other valvular lesions. Which of the following most likely contributed to the development of his cardiac lesion?

A. Congenital bicuspid aortic valve

B. Diabetes mellitus

C. Occult rheumatic heart disease

D. Underlying connective tissue disease

E. None of the above

65. A 63-year-old man presents with new-onset exertional syncope and is found to have aortic stenosis. In counseling the patient, you tell him that your therapeutic recommendation is based on the observation that untreated patients with his presentation have a predicted average life span of:

A. 5 years

B. 4 years

C. 3 years

D. 2 years

E. 1 year

66. Which of the following physical examination findings suggests severe aortic regurgitation?

A. Corrigan’s pulse

B. Pulsus alternans

C. Pulsus bigeminus

D. Pulsus paradoxus

E. Pulsus parvus et tardus

67. A 41-year-old Somali woman is seen in clinic for onset of hemoptysis in the sixth month of her pregnancy. This is her fourth pregnancy and the others were uncomplicated, though she was 35 years old at the birth of her last child. Prior to this, she had been healthy. She reports mild dyspnea beginning at the fourth month of her pregnancy with onset of mild leg swelling shortly thereafter that she attributed to her pregnancy. The dyspnea has become severe, and she is now limited to walking around her house. She began to cough small amounts of bloody sputum 5 days ago. She had neither fever nor purulent sputum and has not responded to a course of antibiotics prescribed by her obstetrician. Physical examination is notable for a normal temperature, heart rate of 110 beats/min, blood pressure of 108/60 mmHg, and oxygen saturation of 91% on room air. No source of bleeding is seen in her nares or oropharynx. Her lungs have diffuse crackles, and cardiac examination shows moderately elevated neck veins, a regular heart rhythm, a loud second heart sound, and a low-pitched diastolic rumble heard best at the apex. The abdomen has a gravid uterus, and 1+ lower extremity edema is present. Which of the following is most likely to demonstrate the cause of her symptoms?

A. Bronchoscopy

B. Chest CT with contrast

C. Echocardiogram

D. Right heart catheterization

E. Upper airway inspection by an otolaryngologist

68. In the patient described in question 67, which of the following should be prescribed at her visit to alleviate her symptoms?

A. Benazepril

B. Digoxin

C. Furosemide

D. Heparin

E. Levofloxacin

69. Which of the following patients with echocardiographic evidence of significant mitral regurgitation has the best indication for surgery with the most favorable likelihood of a positive outcome?

A. A 52-year-old man with an ejection fraction of 25%, NYHA class III symptoms, and a left-ventricular end-systolic dimension of 60 mm

B. A 54-year-old man with an ejection fraction of 30%, NYHA class II symptoms, and pulmonary hypertension

C. A 63-year-old man in sinus rhythm without symptoms, an ejection fraction of 65%, and a normal right heart catheterization

D. A 66-year-old man without symptoms, an ejection fraction of 50%, and left-ventricular end-systolic dimension of 45 mm

E. A 72-year-old asymptomatic woman with newly discovered atrial fibrillation, ejection fraction of 60%, and end-systolic dimension of 35 mm

70. All of the following are potential causes of tricuspid regurgitation EXCEPT:

A. Congenital heart disease

B. Infective endocarditis

C. Inferior wall myocardial infarction

D. Pulmonary arterial hypertension

E. Rheumatic heart disease

F. All of the above will cause tricuspid regurgitation

71. All the following are true about cardiac valve replacement EXCEPT:

A. Bioprosthetic valve replacement is preferred to mechanical valve replacement in younger patients because of the superior durability of the valve.

B. Bioprosthetic valves have a low incidence of thromboembolic complications.

C. The risk of thrombosis with mechanical valve replacement is higher in the mitral position than in the aortic position.

D. Mechanical valves are relatively contraindicated in patients who wish to become pregnant.

E. Double-disk tilting mechanical prosthetic valves offer superior hemodynamic characteristics over single-disk tilting valves.

72. Which of the following infectious agents have been associated with the development of inflammatory myocarditis?

A. Coxsackievirus

B. Diphtheria

C. Q fever

D. Trypanosoma cruzi

E. All of the above

73. All of the following are risk factors for the development of peripartum cardiomyopathy EXCEPT:

A. Advanced maternal age

B. Malnutrition

C. Primiparity

D. Twin pregnancy

E. Use of tocolytics

74. A 67-year-old man with a long history of alcohol abuse presents with findings consistent with left ventricular failure including pulmonary edema and congestion. He undergoes right heart catheterization and left heart catheterization. No significant coronary artery disease is found. Which of the following right heart catheterization numbers (see Table 74) would support a diagnosis of beriberi heart disease?

TABLE 74

Two days after a myocardial infarction, a client begins reporting orthopnea and dyspnea

75. A 20-year-old basketball player is seen for evaluation prior to beginning another season of competitive sports. A harsh systolic murmur is heard at the left lower sternal border. Which of the following maneuvers will enhance this murmur if hypertrophic cardiomyopathy is the underlying cause?

A. Hand grip

B. Leaning forward while sitting

C. Lying left side down

D. Squatting

E. Valsalva maneuver

76. A 62-year-old woman presents to your office with dyspnea of 4 months duration. She has a history of monoclonal gammopathy of unclear significance (MGUS) and has been lost to follow-up for the past 5 years. She is able to do only minimal activity before she has to rest but has no symptoms at rest. She has developed orthopnea but denies paroxysmal nocturnal dyspnea. She complains of fatigue, lightheadedness, and lower extremity swelling. On examination, blood pressure is 110/90 mmHg and heart rate is 94 beats/min. Jugular venous pressure is elevated, and the jugular venous wave does not fall with inspiration. An S3 and S4 are present, as well as a mitral regurgitation murmur. The point of maximal impulse is not displaced. Abdominal examination is significant for ascites and a large, tender, pulsatile liver. Chest radiograph shows bilateral pulmonary edema. An electrocardiogram shows an old left bundle branch block. Which clinical features differentiate constrictive pericarditis from restrictive cardiomyopathy?

A. Elevated jugular venous pressure

B. Kussmaul’s sign

C. Narrow pulse pressure

D. Pulsatile liver

E. None of the above

77. You are evaluating a new patient in the clinic. The 25-year-old patient was diagnosed with “heart failure” in another state and has since relocated. He has New York Heart Association class II symptoms and denies angina. He presents for evaluation and management. The patient has been wheelchair bound for many years and has severe scoliosis. He has no family history of hyperlipidemia. His physical examination is notable for bilateral lung crackles, an S3, and no cyanosis. An electrocardiogram (ECG) is obtained in the clinic and shows tall R waves in V1 and V2 with deep Qs in V5 and V6. An echocardiogram reports severe global left ventricular dysfunction with reduced ejection fraction. What is the most likely diagnosis?

A. Amyotrophic lateral sclerosis

B. Atrial septal defect

C. Chronic thromboembolic disease

D. Duchenne’s muscular dystrophy

E. Ischemic cardiomyopathy

78. A 35-year-old woman with a history of tobacco abuse presents to the emergency department because of severe chest pain radiating to both arms. The pain began 8 hours ago and is worse with inspiration. She has been unable to lie down as this markedly exacerbates the pain, but she feels better with sitting forward. Examination is notable for a heart rate of 96 beats/min, blood pressure of 145/78 mmHg, and oxygen saturation of 98%. Lungs are clear and a friction rub with three components is audible and is best heard at the left lower sternal border. Which of the following are most likely to be found on her ECG?

A. Diffusely inverted T waves in the precordial leads

B. PR elevation in leads II, III, and aVF

C. Sinus tachycardia

D. ST-segment elevation in I, aVL, and V2–V6 with upward concavity and reciprocal depressions in aVR

E. ST-segment elevation V1–V6 with convex curvature and reciprocal depressions in aVR

79. Which of the following statements is true regarding pulsus paradoxus?

A. It consists of a greater than 15 mmHg increase in systolic arterial pressure with inspiration.

B. It may be found in patients with severe obstructive lung disease.

C. It is the reversal of a normal phenomenon during inspiration.

D. It results from right ventricular distention during expiration resulting in compression of the left ventricular volume and subsequent reduction in systolic pulse pressure.

E. All of the above are true.

80. Which of the following are features of Beck’s triad in cardiac tamponade?

A. Hypotension, electrical alternans, prominent x-descent in neck veins

B. Hypotension, muffled heart sounds, electrical alternans

C. Hypotension, muffled heart sounds, jugular venous distention

D. Kussmaul’s sign, hypotension, muffled heart sounds

E. Muffled heart sounds, hypotension, friction rub

81. A 35-year-old woman is admitted to the hospital with malaise, weight gain, increasing abdominal girth, and edema. The symptoms began about 3 months ago and gradually progressed. The patient reports an increase in waist size of approximately 15 cm. The swelling in her legs has gotten increasingly worse such that she now feels her thighs are swollen as well. She has dyspnea on exertion and two-pillow orthopnea. She has a past history of Hodgkin’s disease diagnosed at age 18. She was treated at that time with chemotherapy and mediastinal irradiation. On physical examination, she has temporal wasting and appears chronically ill. Her current weight is 96 kg, which reflects an increase of 11 kg over the past 3 months. Her vital signs are normal. Her jugular venous pressure is approximately 16 cm, and the neck veins do not collapse on inspiration. Heart sounds are distant. There is a third heart sound heard shortly after aortic valve closure. The sound is short and abrupt and is heard best at the apex. The liver is enlarged and pulsatile. Ascites is present. There is pitting edema extending throughout the lower extremities and onto the abdominal wall. Echocardiogram shows pericardial thickening, dilatation of the inferior vena cava and hepatic veins, and abrupt cessation of ventricular filling in early diastole. Ejection fraction is 65%. What is the best approach for treatment of this patient?

A. Aggressive diuresis only

B. Cardiac transplantation

C. Mitral valve replacement

D. Pericardial resection

E. Pericardiocentesis

82. A 19-year-old previously healthy hockey player is defending the goal when he is hit in the left chest with a hockey puck. He immediately collapses to the ice. His coach runs to his side and finds him unresponsive and without a pulse. Which of the following is most likely responsible for this syndrome?

A. Aortic rupture

B. Cardiac tamponade

C. Commotio cordis

D. Hypertrophic cardiomyopathy

E. Tension pneumothorax

83. Which of the following is the most common cause of native valve infective endocarditis in the community?

A. Coagulase-negative staphylococci

B. Coagulase-positive staphylococci

C. Enterococci

D. Fastidious gram-negative coccobacilli

E. Non-enterococcal streptococci

84. All of the following are minor criteria in the Duke criteria for the clinical diagnosis of infective endocarditis EXCEPT:

A. Immunologic phenomena (glomerulonephritis, Osler nodes, Roth spots)

B. New valvular regurgitation on transthoracic echocardiogram

C. Predisposing condition (heart condition, intravenous drug use)

D. Temperature >38.0°C

E. Vascular phenomena (arterial emboli, septic pulmonary emboli, Janeway lesions, and so on)

85. Which of the following patients should receive antibiotic prophylaxis to prevent infective endocarditis?

A. A 23-year-old woman with known mitral valve prolapse undergoing a gingival surgery

B. A 24-year-old woman who had an atrial septal defect completely corrected 22 years ago who is undergoing elective cystoscopy for painless hematuria

C. A 30-year-old man with a history of intravenous drug use and prior endocarditis undergoing operative drainage of a prostatic abscess

D. A 45-year-old man who received a prosthetic mitral valve 5 years ago undergoing routine dental cleaning

E. A 63-year-old woman who received a prosthetic aortic valve 2 years ago undergoing screening colonoscopy

86. A 38-year-old homeless man presents to the emergency department with a transient ischemic attack characterized by a facial droop and left arm weakness lasting 20 minutes and left upper quadrant pain. He reports intermittent subjective fevers, diaphoresis, and chills for the past 2 weeks. He has had no recent travel or contact with animals. He has taken no recent antibiotics. Physical examination reveals a slightly distressed man with disheveled appearance. His temperature is 38.2°C, heart rate is 90 beats/min, and blood pressure is 127/74 mmHg. He has poor dentition. Cardiac examination reveals an early diastolic murmur over the left third intercostal space. His spleen is tender and 2 cm descended below the costal margin. He has tender painful red nodules on the tips of the third finger of his right hand and on the fourth finger of his left hand that are new. He has nits evident on his clothes consistent with body louse infection. His white blood cell count is 14,500/μL with 5% band forms and 93% polymorphonuclear cells. Blood cultures are drawn followed by empirical vancomycin therapy. These cultures remain negative for growth 5 days later. He remains febrile but hemodynamically stable but does develop a new lesion on his toe similar to those on his fingers on hospital day 3. A transthoracic echocardiogram reveals a 1-cm mobile vegetation on the cusp of his aortic valve and moderate aortic regurgitation. A CT scan of the abdomen shows an enlarged spleen with wedge-shaped splenic and renal infarctions. What test should be sent to confirm the most likely diagnosis?

A. Bartonella serology

B. Epstein-Barr virus (EBV) heterophile antibody

C. HIV polymerase chain reaction (PCR)

D. Peripheral blood smear

E. Q fever serology

87. In a patient with bacterial endocarditis, which of the following echocardiographic lesions is most likely to lead to embolization?

A. 5-mm mitral valve vegetation

B. 5-mm tricuspid valve vegetation

C. 11-mm aortic valve vegetation

D. 11-mm mitral valve vegetation

E. 11-mm tricuspid valve vegetation

88. A patient is admitted with fevers, malaise, and diffuse joint pains. His initial blood cultures reveal methicillin-resistant Staphylococcus aureus (MRSA) in all culture bottles. He has no arthritis on examination, and his renal function is normal. echocardiogram shows a 5-mm vegetation on the aortic valve. He is initiated on IV vancomycin at 15 mg/kg every 12 hours. Four days later, the patient remains febrile, and cultures remain positive for MRSA. In addition to a search for embolic foci of infection, which of the following changes would you make to his treatment regimen?

A. No change

B. Add gentamicin

C. Add rifampin

D. Check the vancomycin serum peak and trough levels and consider tid dosing

E. Discontinue vancomycin, start daptomycin

89. Which of the following is the most common clinical presentation of acute rheumatic fever (ARF)?

A. Carditis

B. Chorea

C. Erythema marginatum

D. Polyarthritis

E. Subcutaneous nodules

90. A 19-year-old recent immigrant from Ethiopia comes to your clinic to establish primary care. She currently feels well. Her past medical history is notable for a recent admission to the hospital for new-onset atrial fibrillation. As a child in Ethiopia, she developed an illness that caused uncontrolled flailing of her limbs and tongue lasting approximately 1 month. She also has had three episodes of migratory large-joint arthritis during her adolescence that resolved with pills that she received from the pharmacy. She is currently taking metoprolol and warfarin and has no known drug allergies. Physical examination reveals an irregularly irregular heartbeat with normal blood pressure. Her point of maximal impulse (PMI) is most prominent at the midclavicular line and is normal in size. An early diastolic rumble and a 3/6 holosystolic murmur are heard at the apex. A soft early diastolic murmur is also heard at the left third intercostal space. You refer her to a cardiologist for evaluation of valve replacement and echocardiography. What other intervention might you consider at this time?

A. Glucocorticoids

B. Daily aspirin

C. Daily doxycycline

D. Monthly penicillin G injections

E. Penicillin G injections as needed for all sore throats

91. All of the following statements regarding cardiogenic shock are true EXCEPT:

A. Approximately 80% of cases of cardiogenic shock complicating acute myocardial infarction are attributable to acute severe mitral regurgitation.

B. Cardiogenic shock is more common in ST-segment elevation than non–ST-segment elevation myocardial infarction.

C. Cardiogenic shock is uncommon in inferior wall myocardial infarction.

D. Cardiogenic shock may occur in the absence of significant coronary stenosis.

E. Pulmonary capillary wedge pressure is elevated in cardiogenic shock.

92. Aortic counterpulsation with an intraaortic balloon pump has which of the following as an advantage over therapy with infused vasopressors or inotropes in a patient with acute ST-segment elevation myocardial infarction and cardiogenic shock?

A. Increased heart rate

B. Increased left ventricular afterload

C. Lower diastolic blood pressure

D. Not contraindicated in acute aortic regurgitation

E. Reduced myocardial oxygen consumption

93. Which of the following is the most common electrical mechanism to explain sudden cardiac death?

A. Asystole

B. Bradycardia

C. Pulseless electrical activity (PEA)

D. Pulseless ventricular tachycardia (PVT)

E. Ventricular fibrillation

94. All of the following statements regarding successful resuscitation from sudden cardiac death are true EXCEPT:

A. Advanced age does not affect the likelihood of immediate resuscitation, only the probability of hospital discharge.

B. After out-of-hospital cardiac arrest, survival rates are approximately 25% if defibrillation is administered after 5 minutes.

C. If the initial rhythm in an out-of-hospital cardiac arrest is pulseless ventricular tachycardia, the patient has a higher probability of survival than asystole.

D. Prompt CPR followed by prompt defibrillation improves outcomes in all settings.

E. The probability of survival from cardiac arrest is higher if the event takes place in a public setting than at home.

95. A 32-year-old man is evaluated at a routine clinic visit for coronary risk factors. He is healthy and reports no tobacco use, his systemic blood pressure is normal, and he does not have diabetes. His family history is notable for high cholesterol in his mother and maternal grandparents. Physical examination shows tendon xanthomas. A fasting cholesterol is notable for a low-density lipoprotein cholesterol (LDL-C) of 387 mg/dL. Which of the following is the most likely genetic disorder affecting this individual?

A. Autosomal dominant hypercholesterolemia

B. Familial defective apoB-100

C. Familial hepatic lipase deficiency

D. Familial hypercholesterolemia

E. Lipoprotein lipase deficiency

96. All of the following are potential causes of elevated LDL EXCEPT:

A. Anorexia nervosa

B. Cirrhosis

C. Hypothyroidism

D. Nephrotic syndrome

E. Thiazide diuretics

97. A 16-year-old male is brought to your clinic by his parents due to concern about his weight. He has not seen a physician for many years. He states that he has gained weight due to inactivity and that he is less active because of exertional chest pain. He takes no medications. He was adopted and his parents do not know the medical history of his biological parents. Physical examination is notable for stage 1 hypertension and body mass index of 30 kg/m2. He has xanthomas on his hands, heels, and buttocks. Laboratory testing shows a low-density lipoprotein (LDL) of 210 mg/dL, creatinine of 0.7 mg/dL, total bilirubin of 3.1 mg/dL, haptoglobin below 6 mg/dL, and a glycosylated hemoglobin of 6.7%. You suspect a hereditary lipoproteinemia due to the clinical and laboratory findings. Which test would be diagnostic of the primary lipoprotein disorder in this patient?

A. Congo red staining of xanthoma biopsy

B. CT scan of the liver

C. Family pedigree analysis

D. Gas chromatography

E. LDL receptor function in skin biopsy

98. Your 60-year-old patient with a monoclonal gammopathy of unclear significance presents for a follow-up visit and to review recent laboratory data. His creatinine is newly elevated to 2.0 mg/dL, potassium is 3.7 mg/dL, calcium is 12.2 mg/dL, low-density lipoprotein (LDL) is 202 mg/dL, and triglycerides are 209 mg/dL. On further questioning he reports 3 months of swelling around the eyes and “foamy” urine. On examination, he has anasarca. Concerned for multiple myeloma and nephrotic syndrome, you order a urine protein/creatinine ratio, which returns at 14:1. Which treatment option would be most appropriate to treat his lipid abnormalities?

A. Cholesterol ester transfer protein inhibitor

B. Dietary management

C. HMG-CoA reductase inhibitors

D. Lipid apheresis

E. Niacin and fibrates

99. A 48-year-old white man is seen in the clinic for a routine physical examination. He reports no complaints. Examination shows a blood pressure of 134/82 mmHg with a normal heart rate. BMI is 31 kg/m2. The remainder of his physical examination is normal. Which of the following is true regarding lifestyle modification?

A. Brisk walking for as little as 10 minutes, 4 days per week will lower his blood pressure to within the normal range.

B. Dietary NaCl restriction of less than 6 g per day will reduce his blood pressure.

C. Lifestyle modification will have no effect on his blood pressure.

D. Reduction of alcohol consumption to three or fewer drinks per day will decrease his blood pressure.

E. Weight loss of approximately 9 kg can be expected to bring his blood pressure to within the normal limit.

100. A 46-year-old white female presents to your office with concerns about her diagnosis of hypertension 1 month previously. She asks you about her likelihood of developing complications of hypertension, including renal failure and stroke. She denies any past medical history other than hypertension and has no symptoms that suggest secondary causes. She currently is taking hydrochlorothiazide 25 mg/d. She smokes half a pack of cigarettes daily and drinks alcohol no more than once per week. Her family history is significant for hypertension in both parents. Her mother died of a cerebrovascular accident. Her father is alive but has coronary artery disease and is on hemodialysis. Her blood pressure is 138/90 mmHg. Body mass index is 23. She has no retinal exudates or other signs of hypertensive retinopathy. Her point of maximal cardiac impulse is not displaced but is sustained. Her rate and rhythm are regular and without gallops. She has good peripheral pulses. An electrocardiogram reveals an axis of –30 degrees with borderline voltage criteria for left ventricular hypertrophy. Creatinine is 1.0 mg/dL. Which of the following items in her history and physical examination is a risk factor for a poor prognosis in a patient with hypertension?

A. Family history of renal failure and cerebrovascular disease

B. Persistent elevation in blood pressure after the initiation of therapy

C. Ongoing tobacco use

D. Ongoing use of alcohol

E. Presence of left ventricular hypertrophy on ECG

101. A 28-year-old female has hypertension that is difficult to control. She was diagnosed at age 26. Since that time she has been on increasing amounts of medication. Her current regimen consists of labetalol 1000 mg bid, lisinopril 40 mg qd, clonidine 0.1 mg bid, and amlodipine 5 mg qd. On physical examination she appears to be without distress. Blood pressure is 168/100 mmHg, and heart rate is 84 beats/min. Cardiac examination is unremarkable, without rubs, gallops, or murmurs. She has good peripheral pulses and has no edema. Her physical appearance does not reveal any hirsutism, fat maldistribution, or abnormalities of genitalia. Laboratory studies reveal a potassium of 2.8 meq/dL and a serum bicarbonate of 32 meq/dL. Fasting blood glucose is 114 mg/dL. What is the likely diagnosis?

A. Congenital adrenal hyperplasia

B. Fibromuscular dysplasia

C. Cushing’s syndrome

D. Conn’s syndrome

E. Pheochromocytoma

102. What is the best way to diagnose this disease in question 101?

A. Renal vein renin levels

B. 24-hour urine collection for metanephrines

C. Magnetic resonance imaging of the renal arteries

D. 24-hour urine collection for cortisol

E. Plasma aldosterone/renin ratio

103. Which of the following patients with aortic dissection or hematoma is best managed without surgical therapy?

A. A 74-year-old male with a dissection involving the root of the aorta.

B. A 45-year-old female with a dissection involving the aorta distal to the great vessel origin but cephalad to the renal arteries.

C. A 58-year-old male with aortic dissection involving the distal aorta and the bilateral renal arteries.

D. A 69-year-old male with an intramural hematoma within the aortic root.

E. All of the above patients require surgical management of their aortic disease.

104. A 68-year-old male presents to your office for routine follow-up care. He reports that he is feeling well and has no complaints. His past medical history is significant for hypertension and hypercholesterolemia. He continues to smoke a pack of cigarettes daily. He is taking chlorthalidone 25 mg daily, atenolol 25 mg daily, and pravastatin 40 mg nightly. Blood pressure is 133/85 mmHg, and heart rate is 66 beats/min. Cardiac and pulmonary examinations are unremarkable. A pulsatile abdominal mass is felt just to the left of the umbilicus and measures approximately 4 cm. You confirm the diagnosis of abdominal aortic aneurysm by CT imaging. It is located infrarenally and measures 4.5 cm. All the following are true about the patient’s diagnosis EXCEPT:

A. The 5-year risk of rupture of an aneurysm of this size is 1–2%.

B. Surgical or endovascular intervention is warranted because of the size of the aneurysm.

C. Infrarenal endovascular stent placement is an option if the aneurysm experiences continued growth in light of the location of the aneurysm infrarenally.

D. Surgical or endovascular intervention is warranted if the patient develops symptoms of recurrent abdominal or back pain.

E. Surgical or endovascular intervention is warranted if the aneurysm expands beyond 5.5 cm.

105. A 32-year-old female is seen in the emergency department for acute shortness of breath. A helical CT shows no evidence of pulmonary embolus, but incidental note is made of dilatation of the ascending aorta to 4.3 cm. All the following are associated with this finding EXCEPT:

A. Syphilis

B. Takayasu’s arteritis

C. Giant cell arteritis

D. Rheumatoid arthritis

E. Systemic lupus erythematosus

106. A 68-year-old man with a history of coronary artery disease is seen in his primary care clinic for complaint of cough with sputum production. His care provider is concerned about pneumonia, so a chest radiograph is ordered. On the chest radiograph, the aorta appears tortuous with a widened mediastinum. A contrast-enhanced CT of the chest confirms the presence of a descending thoracic aortic aneurysm measuring 4 cm with no evidence of dissection. What is the most appropriate management of this patient?

A. Consult interventional radiology for placement of an endovascular stent.

B. Consult thoracic surgery for repair.

C. No further evaluation is needed.

D. Perform yearly contrast-enhanced chest CT and refer for surgical repair when the aneurysm size is greater than 4.5 cm.

E. Treat with beta blockers, perform yearly contrast-enhanced chest CT, and refer for surgical repair if the aneurysm grows more than 1 cm/year.

107. A 37-year-old woman with no significant past medical history except for a childhood murmur is evaluated for severe pain of sudden onset in her right lower extremity. Examination is notable for a young, uncomfortable woman with normal vital signs except for a heart rate of 110 beats/min. Right leg has pallor distal to the right knee and is cold to the touch, and the dorsalis pedis pulse is absent. Which of the following studies is likely to diagnose the underlying reason for the patient’s presentation?

A. Angiography of right lower extremity

B. Blood cultures

C. Echocardiogram with bubble study

D. Serum c-ANCA

E. Venous ultrasound of right upper extremity

108. Doppler echocardiography is most useful for diagnosis of which of the following cardiac lesions?

A. Determination of cardiac mass in a patient with an audible “plop” on examination

B. Determination of left ventricular ejection fraction in a patient with a history of myocardial infarction

C. Diagnosis of myocardial ischemia in a patient with atypical chest pain

D. Diagnosis of pericardial effusion

E. Diastolic filling assessment in a patient with suspected heart failure with preserved ejection fraction

1. The answer is D.

(Chap. 5) Shortness of breath, or dyspnea, is a common presenting complaint in primary care. However, dyspnea is a complex symptom and is defined as the subjective experience of breathing discomfort that includes components of physical as well as psychosocial factors. A significant body of research has been developed regarding the language by which a patient describes dyspnea with certain factors being more common in specific diseases. Individuals with airways diseases (asthma, chronic obstructive pulmonary disease [COPD]) often describe air hunger, increased work of breathing, and the sensation of being unable to get a deep breath because of hyperinflation. In addition, individuals with asthma often complain of a tightness in the chest. Individuals with cardiac causes of dyspnea also describe chest tightness and air hunger but do not have the same sensation of being unable to draw a deep breath or have increased work of breathing. A careful history will also lead to further clues regarding the cause of dyspnea. Nocturnal dyspnea is seen in congestive heart failure or asthma, and orthopnea is reported in heart failure, diaphragmatic weakness, and asthma that is triggered by esophageal reflux. When discussing exertional dyspnea, it is important to assess if the dyspnea is chronic and progressive or episodic. Whereas episodic dyspnea is more common in myocardial ischemia and asthma, COPD and interstitial lung diseases present with a persistent dyspnea. Platypnea is a rare presentation of dyspnea in which a patient is dyspneic in the upright position and feels improved with lying flat. On physical examination of a patient with dyspnea, the physician should observe the patient’s ability to speak and the use of accessory muscle or preference of the tripod position. As part of vital signs, a pulsus paradoxus may be measured with a value of greater than 10 mmHg common in asthma and COPD. Pulsus paradoxus greater than 10 mmHg may also occur in pericardial tamponade. Lung examination may demonstrate decreased diaphragmatic excursion, crackles, or wheezes that allow one to determine the cause of dyspnea. Further workup may include pulmonary function testing, chest radiography, chest CT, electrocardiography, echocardiography, or exercise testing, among others, to ascertain the cause of dyspnea.

2. The answer is D.

(Chap. 6) When a patient presents for evaluation of hypoxia, it is important to consider the underlying mechanism of hypoxia in order to determine the etiology. The primary causes of hypoxia are related to respiratory disease and include ventilation/perfusion (V/Q) mismatch, hypoventilation, and intrapulmonary right-to-left shunting. Causes of hypoxia outside of the respiratory system include intracardiac right-to-left shunting, high-altitude hypoxia, anemic hypoxia, circulatory hypoxia, and carbon monoxide poisoning. In this patient, the mechanism of hypoxia can be narrowed to two possibilities—intracardiac versus intrapulmonary right-to-left shunting—quite easily because the patient failed to correct his hypoxia in response to 100% oxygen. The history of platypnea and orthodeoxia is suggestive that the likely cause is intrapulmonary rather than intracardiac shunting. The finding of a possible lung nodule on chest radiographs in the lower lung fields also is supportive of a pulmonary cause of shunting through an arteriovenous malformation, which can appear as a lung nodule on chest x-ray. An intracardiac right-toleft shunt is caused by congenital cardiac malformations and Eisenmenger syndrome. If there was an intracardiac cause of shunt, the cardiac examination would be expected to demonstrate a murmur and/or evidence of pulmonary hypertension.

V/Q mismatch is the most common cause of hypoxia and results from perfusion of areas of the lung that receive limited ventilation. Examples of V/Q mismatch include asthma, chronic obstructive pulmonary disease, and pulmonary embolus. Hypoxia caused by V/Q mismatch can be corrected with supplemental oxygen. Hypoventilation can be caused by multiple causes, including acute respiratory depression or chronic respiratory failure with elevations in PaCO2. Hypoxia caused by hypoventilation is also correctable with oxygen but frequently has a normal alveolar–arterial oxygen gradient.

Causes of hypoxia outside the respiratory system are less common. High-altitude hypoxia becomes apparent when individuals travel to elevations greater than 3000 m. Anemic hypoxia is not associated with a decrease in PaO2, but a decrease in hemoglobin does cause decreased oxygen-carrying capacity in the blood and relative tissue hypoxia if severe. Circulatory hypoxia refers to tissue hypoxia that occurs because of a decrease cardiac output that leads to greater tissue extraction of oxygen. As a result, the venous partial pressure of oxygen is reduced, and there is an increased arterial-mixed venous oxygen gradient.

3. The answer is C.

(Chap. 6) In the evaluation of cyanosis, the first step is to differentiate central from peripheral cyanosis. In central cyanosis, because the etiology is either reduced oxygen saturation or abnormal hemoglobin, the physical findings include bluish discoloration of both mucous membranes and skin. In contrast, peripheral cyanosis is associated with normal oxygen saturation but slowing of blood flow and an increased fraction of oxygen extraction from blood; subsequently, the physical findings are present only in the skin and extremities. Mucous membranes are spared. Peripheral cyanosis is commonly caused by cold exposure with vasoconstriction in the digits. Similar physiology is found in Raynaud’s phenomenon. Peripheral vascular disease and deep venous thrombosis result in slowed blood flow and increased oxygen extraction with subsequent cyanosis. Methemoglobinemia causes abnormal hemoglobin that circulates systemically. Consequently, the cyanosis associated with this disorder is systemic. Other common causes of central cyanosis include severe lung disease with hypoxemia, right-to-left intracardiac shunting, and pulmonary arteriovenous malformations.

4. The answer is B.

(Chaps. 9, 40) Pulmonary hypertension is associated with a loud second heart sound that is heard to be louder than the first heart sound at the cardiac base. In idiopathic pulmonary arterial hypertension, there is no associated congenital lesion, such as atrial septal defect (ASD). In ASD, the components of the second heart sound, aortic and pulmonic valve closure, do not alter their timing with respect to respiratory cycle and are always widely split, and thus are described as “fixed split.” In idiopathic pulmonary arterial hypertension, the components of the second heart sound are nearly superimposed and loud; often there is little respiratory variation. The soft systolic murmur at the left lower sternal border of tricuspid regurgitation is nearly always present in pulmonary hypertension of all etiologies. Idiopathic pulmonary arterial hypertension, by definition, is not associated with a parenchymal lung disease such as emphysema. Patients with idiopathic pulmonary arterial hypertension should not have physical findings associated with chronic airways disease.

5. The answer is A.

(Chap. 9) The patient is very likely to have pericardial tamponade from metastatic cancer as suggested by her elevated neck veins, heart shadow shape and size, and predisposing condition. Because of the exaggerated interventricular dependence, the normal (<10 mmHg) fall in systemic blood pressure with inspiration is exaggerated (often >15 mmHg) with cardiac tamponade. This is referred to as pulsus paradoxus, though it is in fact an augmentation of a normal finding. Kussmaul’s sign, or a lack of fall of the jugular venous pressure with inspiration, usually denotes a lack of compliance in the right ventricle, as seen most frequently in constrictive pericarditis, though it may be found in restrictive cardiomyopathy or massive pulmonary embolism. A rapid y-descent, which follows the peak of the v wave, of jugular venous pressure tracing is indicative of cardiac tamponade. Pulsus parvus et tardus, or small and slow arterial pulsation, is a late finding in aortic stenosis. Late diastolic murmur and opening snap is found in mitral stenosis.

6. The answer is A.

(Chap. 9) A fourth heart sound indicates left ventricular presystolic expansion and is common among patients in whom active atrial contraction is important for ventricular filling. A fourth heart sound is not found in atrial fibrillation. An irregular heart rate is characteristic of atrial fibrillation. The irregular rate is often characterized as “irregularly irregular.” A third heart sound occurs during the rapid filling phase of ventricular diastole and indicates heart failure. Reversed splitting of the second heart sound occurs with left bundle branch block, as this patient has. Finally, pulsus alternans is beat-to-beat variability in pulse amplitude. It is present when only every other Korotkoff sound is audible as the cuff pressure is lowered slowly. It is thought to be due to cyclic changes in intracellular calcium and action potential duration and is associated with severe left ventricular failure.

7. The answer is A.

(Chap. 9) The presentation of this patient is consistent with the diagnosis of acute valvular dysfunction due to infective endocarditis. The presence of a widened pulse pressure and diastolic murmur heard best along the lower sternal border suggests aortic regurgitation. Panel C of Figure 9-2 shows a typical bisferiens pulse that is characteristic of aortic regurgitation. With a bisferiens pulse, there are two distinct pulsations that can be palpated with systole. The initial pulse represents an exaggerated percussion wave reflecting the increased stroke volume that occurs in aortic regurgitation, with the second peak reflecting the tidal, or anacrotic, wave.

Infective endocarditis causes loss of valvular integrity and acutely causes valvular regurgitation. Of the other options, both mitral regurgitation and tricuspid regurgitation (choice E) would cause systolic and not diastolic murmurs. A hyperkinetic pulse may occur in these conditions, particularly if associated with fever or sepsis. With a hyperkinetic pulse the usual dichrotic notch is more pronounced, as seen in panel E of the figure. Mitral stenosis causes a diastolic murmur but is not a common lesion associated with infective endocarditis, unless underlying valvular stenosis was present prior to acquiring the infection. It is not associated with a bisferiens pulse. Aortic stenosis is associated with pulsus parvus et tardus, with a delayed and prolonged carotid upstroke as shown in panel B of the figure. Aortic stenosis has an associated harsh crescendo-decrescendo systolic murmur.

8. The answer is A.

(Chap. 9) Peripheral arterial disease (PAD) affects 5–8% of Americans, with increasing incidence with age. Over the age of 65, the incidence of PAD rises to between 12% and 20%. The primary symptom of PAD is claudication. As this patient describes, claudication occurs with ambulation and is often described as a crampy to aching pain that is relieved with rest. On physical examination, those with PAD often have diminished peripheral pulses, delayed capillary refill, and hair loss in the distal extremities. The skin is often cool to the touch with a thin, shiny appearance. In severe PAD, pain in the extremities occurs at rest. Diagnosis of PAD can be suggested by these findings and should be documented by determination of the ankle-brachial index (ABI), as physical examination alone is insufficient to diagnose PAD. Although lack of a palpable pulse suggests critical ischemia, it is not diagnostic. To perform an ABI, blood pressures are determined in the arm and the lower extremities. Either the dorsalis pedis or posterior tibial pulses can be used. The ABI is calculated by dividing the ankle systolic pressure by the brachial systolic pressure. A resting ABI less than 0.9 is abnormal, but critical ischemia with rest pain does not occur until the ABI is less than 0.3. In individuals with heavily calcified blood vessels, the ABI can be abnormally elevated (ABI >1.2) when PAD is present. In this situation, toe pressures to determine ABI or employing imaging techniques such as MRI or arteriography should be considered. Lower extremity edema is suggestive of congestive heart failure, not PAD.

9. The answer is F.

(Chap. 9) When a murmur of uncertain cause is identified on physical examination, a variety of physiologic maneuvers can be used to assist in the elucidation of the cause. Commonly used physiologic maneuvers include change with respiration, Valsalva maneuver, position, and exercise. In hypertrophic cardiomyopathy, there is asymmetric hypertrophy of the interventricular septum, which creates a dynamic outflow obstruction. Maneuvers that decrease left-ventricular filling will cause an increase in the intensity of the murmur, whereas those that increase left-ventricular filling will cause a decrease in the murmur. Of the interventions listed, both standing and a Valsalva maneuver will decrease venous return and subsequently decrease left ventricular filling, resulting in an increase in the loudness of the murmur of hypertrophic cardiomyopathy. Alternatively, squatting will increase venous return and thus decrease the murmur. Maximum hand-grip exercise also will result in a decreased loudness of the murmur.

10. The answer is C.

(Chap. 10) Mitral valve prolapse is characterized by a mid-systolic nonejection sound (click) followed by a late systolic murmur that crescendos and terminates with S2. A decrease in venous return induced by standing will move the click closer to S1 and increase the duration of the murmur. Squatting will increase venous return and shorten the duration of the murmur. The murmur of hypertrophic cardiomyopathy behaves in a similar fashion, but there would be no nonejection click, and left ventricular hypertrophy would be expected on electrocardiography (ECG). Aortic stenosis is best heard at the right second intercostal space radiating to the carotid and is crescendo-decrescendo in character. Congenital pulmonic stenosis is crescendo-decrescendo in character and is heard best in the second to third left intercostal space. If severe, there is a parasternal lift right ventricular overload on ECG. Tricuspid regurgitation causes a holosystolic, not mid-systolic, murmur that increases with inspiration.

11. The answer is C.

(Chap. 10) Tricuspid regurgitation and mitral regurgitation (along with ventricular septal defect) cause holosystolic murmurs. These murmurs have their onset with S1 and terminate at or with S2. Whereas tricuspid regurgitation is heard best over the left sternal border, mitral regurgitation is heard best at the apex with radiation to the base or axilla. The onset of a murmur after S1 with a nonejection sound (click) is characteristic of mitral valve prolapse. Amyl nitrate decreases the intensity of mitral regurgitation and ventricular septal defect murmurs. Tricuspid regurgitation increases with inspiration. Wide splitting of S2 is characteristic of ventricular septal defects. Inaudible A2 at the ventricular apex is characteristic of mitral regurgitation. Because of the incompetent tricuspid valve, the murmur of tricuspid regurgitation is associated with prominent c-v waves and a sharp y-descent in the jugular venous pulse.

12. The answer is A.

(Chap. 10) Evaluating the splitting of the aortic (A2) and pulmonic (P2) components of the second heart sound (S2) during auscultation can be diagnostically useful. In normal conditions, P2 follows A2, and the splitting increases during inspiration. Reversed (or paradoxical) splitting of S2, when P2 precedes A2 during expiration (and they come closer together during inspiration), is attributable to a delay in A2 and is characteristic of severe aortic stenosis, hypertrophic obstructive cardiomyopathy, left bundle branch block, right ventricular pacing, or acute myocardial ischemia. Wide splitting of S2 is an accentuation of the physiologic pattern usually caused by delayed pulmonic valve closing (right bundle branch block, pulmonary stenosis, pulmonary hypertension) or early aortic valve closure (severe mitral regurgitation). Fixed splitting (no respiratory variation) with the murmur described is characteristic of an atrial septal defect. This is an important finding because it may be asymptomatic until the third or fourth decade of life and, if undiagnosed, may lead to severe pulmonary hypertension and Eisenmenger syndrome.

13. The answer is B.

(Chap. 11) Left bundle branch, defined by QRS interval greater than 120 milliseconds with typical pattern in V1 and V6, is associated with four conditions: coronary heart disease, hypertensive heart disease, aortic valve disease, and cardiomyopathy. In all cases, the left bundle branch block is associated with increased risk of cardiovascular morbidity and mortality. These conditions share left ventricular pathology. In contrast, right bundle branch block is associated with congenital heart disease, pulmonary vascular disease, and less frequently valvular heart disease.

14. The answer is D.

(Chap. 11) The classic findings of hypokalemia are prominent U waves due to prolonged ventricular repolarization. Scooped ST segments are commonly seen with digoxin toxicity. Low P wave amplitude is found in early hyperkalemia. Prolonged QT intervals are often due to drug toxicity such as tricyclic antidepressant overdose, procainamide, quinidine disopyramide, and phenothiazines. Finally, Osborne waves, or convex elevation of the J point, are found in severe hypothermia and are due to repolarization prolongation.

15. The answer is D.

(Chaps. 11, 20, and 40) The patient presents from a country with likely low rates of treatment for childhood streptococcal infection and was subsequently at high risk for rheumatic heart disease. Her large pulmonary arteries in the absence of parenchymal infiltrates suggests pulmonary hypertension and her ECG shows right ventricular hypertrophy, characterized by a relatively tall R wave in lead V1, or R greater than or equal to S wave. This is highly likely to be due to mitral stenosis. While aortic stenosis and regurgitation are possible causes, these are less likely. Tricuspid stenosis is not associated with right ventricular hypertrophy. Left ventricular systolic failure may cause pulmonary venous hypertension, but more commonly is associated with evidence of left heart failure on examination.

16. The answer is D.

(Chaps. 11 and 41) This ECG shows a short ST segment that is most prominent in V2, V3, V4, and V5. Hypercalcemia, by shortening the duration of repolarization, abbreviates the total time from depolarization through repolarization. This is manifested on the surface ECG by a short QT interval. In this scenario, the hypercalcemia is due to the rhabdomyolysis and renal failure. Fluids and a loop diuretic are an appropriate therapy for hypercalcemia. Hemodialysis is seldom indicated. Hemodialysis is indicated for significant hyperkalemia, which may also develop after rhabdomyolysis, manifest by “tenting” of the T waves or widening of the QRS. Classic ECG manifestations of a pulmonary embolus (S1, Q3, T3 pattern) are infrequent in patients with pulmonary embolism (PE), though the changes may be seen with massive PE. There are no signs of myocardial ischemia on this ECG, which would make coronary catheterization and 18-lead ECG interpretation of low yield.

17. The answer is D.

(Chap. 11) Hyperkalemia leads to partial depolarization of cardiac cells. As a result, there is slowing of the upstroke of the action potential as well as reduced duration of repolarization. The T wave becomes peaked, the RS complex widens and may merge with the T wave (giving a sine-wave appearance), and the P wave becomes shallow or disappears. Prominent U waves are associated with hypokalemia; ST-segment prolongation is associated with hypocalcemia.

18. The answer is C.

(Chaps. 11 and 21) The ECG shows slight right axis deviation and low voltage. These changes are typical of emphysema when the thorax is hyperinflated with air and the flattened diaphragm pulls the heart inferiorly and vertically. An acute central nervous system (CNS) event such as a subarachnoid hemorrhage may cause QT prolongation with deep, wide inverted T waves. Hyperkalemia will cause peaked narrowed T waves or a wide QRS complex. Patients with hypertrophic cardiomyopathy will have left ventricular hypertrophy and widespread deep, broad Q waves.

19. The answer is E.

(Chaps. 11 and 43) This ECG tracing shows the triad of a short PR interval, wide QRS, and delta waves (seen best in leads I, II, and V5) consistent with Wolff-Parkinson-White (WPW) syndrome. Patients with WPW syndrome are commonly diagnosed asymptomatically when an ECG is performed showing the classic findings. Symptoms are due to conduction via an accessory pathway and include tachypalpitations, lightheadedness, syncope, cardiopulmonary collapse, and sudden cardiac death. Life-threatening presentations are usually due to the development of atrial fibrillation or atrial flutter with 1:1 conduction, both of which can precipitate ventricular fibrillation. Unstable angina is mainly associated with ST-segment abnormalities, although conduction abnormalities may be seen. Pulmonary embolism, which may cause hemoptysis and pleuritic chest pain, has nonspecific ECG findings including S1Q3T3 (acute right ventricular failure) or T-wave abnormalities.

20. The answer is E.

(Chap. 11) The limb lead aVR generally has a negative deflection, as the primary vector for ventricular depolarization is directed down and away from this lead. Therefore, in the case of left ventricular hypertrophy the negative deflection, or S wave, would be expected to be larger without an effect on the R wave. There are multiple criteria for diagnosing left ventricular hypertrophy on ECG.

21. The answer is B.

(Chaps. 11 and 43) This ECG tracing shows multifocal atrial tachycardia (MAT), right atrial overload, a superior axis, and poor R-wave progression in the precordial leads. There are varying P-wave morphologies (more than three morphologies) and P-P intervals. MAT is most commonly caused by COPD, but other conditions associated with this arrhythmia include coronary artery disease, congestive heart failure, valvular heart disease, diabetes mellitus, hypokalemia, hypomagnesemia, azotemia, postoperative state, and pulmonary embolism. Anemia, pain, and myocardial ischemia are also causes of tachycardia that should be considered when managing a new tachycardia. These states are usually associated with sinus tachycardia.

22. The answer is E.

(Chap. 13) Although myocardial infarction, stroke, and death are complications that have been reported with cardiac catheterization (all with a frequency of <0.1%), the more common complications are tachy- or bradyarrhythmias, acute renal failure, and vascular complications. Vascular access site bleeding is the most common complication of cardiac catheterization, occurring in 1.5–2% of patients. When catheterization is performed in an emergent fashion for acute myocardial infarction or for hemodynamically unstable patients, the complication rate may rise substantially.

23. The answer is A.

(Chap. 13) Although right heart catheterization is no longer routinely performed at the time of left heart catheterization, there remain important indications for this procedure. These include evaluation of unexplained dyspnea, especially when there is a suspicion of pulmonary hypertension; diagnosis of valvular heart disease such as mitral regurgitation; pericardial disease; right and/or left ventricular dysfunction, particularly for determination of severity; diagnosis of congenital heart disease; and suspected intracardiac shunts. In this case, the patient likely has an atrial septal defect with physical examination findings of a loud, fixed split second heart sound and perhaps associated dyspnea. During right heart catheterization, the pulmonary arterial pressures will be measured to assess for pulmonary hypertension, and venous saturation will be measured at the inferior vena cava, right atrium, right ventricle, and pulmonary artery to assess for evidence of an increase in saturation suggestive of intracardiac shunt. All the other patients described would be more appropriately served with a left heart catheterization and coronary angiogram.

24. The answer is B.

(Chaps. 13 and 40) In the diagnostic algorithm for pulmonary hypertension, the right heart catheterization is important to document the presence and degree of pulmonary hypertension. The right-ventricular systolic pressure (RVSP) on echocardiography provides an estimate of pulmonary arterial pressures, but accurate determination of the RVSP relies on the presence of tricuspid regurgitation and good quality echocardiography. This patient’s body habitus is prohibitive in obtaining good windows for echocardiography. Thus, a right heart catheterization is imperative for documenting pulmonary hypertension, as well as for determining the cause. The right heart catheterization demonstrates an elevated mean arterial pressure, elevated left-ventricular end-diastolic pressure (pulmonary capillary wedge pressure), and elevated mean pulmonary artery pressure. In the presence of a normal cardiac output and an elevated left-ventricular ejection fraction, this is consistent with the diagnosis of diastolic heart failure. Systolic heart failure is associated with similar indices on right heart catheterization, but left-ventricular function is depressed in systolic heart failure. The other causes listed as options are known causes of pulmonary hypertension but would not be expected to cause an increase in the left-ventricular end-diastolic pressure. Obstructive sleep apnea is usually associated only with mild elevations in pulmonary artery pressure. This patient’s BMI puts her at risk for obstructive sleep apnea but would not be responsible for these right heart catheterization values. Both chronic thromboembolic disease and pulmonary arterial hypertension can cause severe elevations in the pulmonary arterial pressure but have a normal left atrial pressure.

25. The answer is B.

(Chap. 15) The tachycardia-bradycardia variant of sick sinus syndrome is associated with an increased risk of thromboembolism, particularly when similar risk factors are present that increase the risk of thromboembolism in patients with atrial fibrillation. Specific risk factors associated with highest risk include age greater than 65 years and prior history of stroke, valvular heart disease, left ventricular dysfunction, or atrial enlargement. Patients with these risk factors should be treated with anticoagulation.

26. The answer is D.

(Chap. 15) Bradycardia is frequently present in trained athletes, particularly at night, where heart rates are usually between 40 and 60 beats/min. While sleep apnea can be associated with bradycardia, no apnea was found in this patient on overnight polysomnography. Other possible causes of bradycardia in this patient such as hypothyroidism have been ruled out. Measurement of free T4 is not indicated with a normal TSH. Pacemaker insertion is not indicated for his normal physiology. Carotid sinus massage is likely to cause further bradycardia. Fatigue is likely due to his stressful job.

27. The answer is E.

(Chap. 15) Sinoatrial dysfunction is often divided into intrinsic disease and extrinsic disease of the node. This is a critical distinction, as extrinsic causes are often reversible and pacemaker placement is not required. Drug toxicity is a common cause of extrinsic, reversible sinoatrial dysfunction, with common culprits including beta blockers, calcium channel blockers, lithium toxicity, narcotics, pentamidine, and clonidine. Hypothyroidism, sleep apnea, hypoxia, hypothermia, and increased intracranial pressure are all reversible forms of extrinsic dysfunction. Radiation therapy can result in permanent dysfunction of the node and therefore is an irreversible, or intrinsic, cause of sinoatrial node dysfunction. In symptomatic patients, pacemaker insertion may be indicated.

28. The answer is B.

(Chap. 15) When there is evidence of sinoatrial node dysfunction, as manifest in this patient with sinus bradycardia, the first approach is to search for reversible causes. In this case, excessive beta blockade is the most likely explanation for his bradycardia and symptoms. Stopping the metoprolol at least temporarily is in order. There are no urgent indications for temporary or permanent pacemaker placement, as he does not have a high-level AV block, syncope, or shock. His heart failure should reverse when his heart rate increases. Although pharmacologic chronotropic stimulation can increase heart rate temporarily, his moderate symptoms suggest that simply waiting for the beta blocker to be metabolized will be adequate. There is no evidence of new infarction or post-infarct angina; thus the patient does not require urgent revascularization. Once the patient is stabilized, the risks and benefits of restarting the beta blocker at a lower dosage may be considered.

29. The answer is C.

(Chap. 15) The patient presents with a classic bulls-eye lesion, or erythema migrans, consistent with Lyme disease. Cardiac conduction abnormalities are common in Lyme disease, often involving the AV node. Temporary pacing may be necessary, but the conduction abnormalities usually resolve. The most common test to diagnose this condition is an ELISA with confirmatory Western blot. Other infectious etiologies can present with heart block such as syphilis and Chagas’ disease, but these would not be associated with the characteristic Lyme rash. Autoimmune and infiltrative diseases may also present with conduction system disease such as ankylosing spondylitis, rheumatoid arthritis, scleroderma, and systemic lupus erythematosus.

30. The answer is B.

(Chap. 15) Second-degree AV block type 1 (Mobitz type 1) is characterized by a progressive lengthening of the PR interval preceding a pause. The pause in this tracing is between the third and fourth QRS complex. First-degree AV block is a slowing of conduction through the AV junction and is diagnosed when the PR interval is greater than 200 milliseconds. Type 2 second-degree AV block is characterized by intermittent failure of conduction of the P wave without changes in the preceding PR or RR intervals. Second-degree AV block type 2 usually occurs in the distal or infra-His conduction systems.

31 and 32. The answers are B and D, respectively.

(Chap. 16) The patient has persistent, non–life-threatening palpitations that distress her enough to seek medical attention. A continuous Holter monitor for 24 hours is appropriate for patients in whom the symptoms happen several times a day in which an event monitor is triggered by the patient when symptoms occur and thus can be worn for a longer period of time. There is no indication of gastrointestinal triggers, so abdominal CT would not be helpful. The atrial premature contractions are uncomplicated, do not require additional diagnostic evaluation at this time, and pose no additional health risk. EP referral is indicated for patients with life-threatening or severe symptoms such as syncope.

33. The answer is C.

(Chap. 16) The patient has physiologic sinus tachycardia related to a pneumothorax, for which he was at risk from his obstructive lung disease and volume-cycled mechanical ventilation. The increased peak inspiratory pressure on the mechanical ventilator is due to the reduced respiratory system compliance from the pneumothorax. Physiologic sinus tachycardia often comes on slowly and responds poorly to carotid sinus massage with gradual return to original rate. Pharmacologic interventions are usually unsuccessful with correction of the underlying cause required for resolution of the tachycardia. In this case, a tension pneumothorax is confirmed by chest radiograph and, with placement of a chest tube, the tachycardia resolves. Other causes of physiologic sinus tachycardia include pain, hyperthyroidism, anxiety, anemia, hypotension, fever, and exercise.

34. The answer is E.

(Chap. 16) Patients at the highest risk for stroke associated with atrial fibrillation include those with a prior history of stroke, TIA, or embolism, and patients with hypertension, diabetes mellitus, congestive heart failure, rheumatic heart disease, LV dysfunction, and marked left atrial dilation of greater than 5.0 cm or age greater than 65 years. Anticoagulation should be strongly considered in these patients. Increased left atrial size is a risk factor for chronic atrial fibrillation.

35. The answer is B.

(Chap. 16) The AFFIRM and RACE trials compared outcomes in survival and thromboembolic events in patients with atrial fibrillation using two treatment strategies: rate control and anticoagulation versus pharmacotherapy to maintain sinus rhythm. There was no difference in events in the two groups, which is thought to be due to the inefficiencies of pharmacotherapy, with over half of patients failing drug therapy, and also the high rates of asymptomatic atrial fibrillation in the sinus rhythm group. Thus, when considering discontinuation of anticoagulation in patients who have maintained sinus rhythm, placing a prolonged ECG monitor is recommended to ensure that asymptomatic atrial fibrillation is not present. Because of the risk of QT prolongation and polymorphic ventricular tachycardia, initiation of dofetilide and sotalol in the hospital is recommended.

36. The answer is C.

(Chap. 16) The patient has atrial flutter, which has a high risk of thromboembolic events and should be treated the same as atrial fibrillation. If atrial flutter has been present for more than 24–48 hours without anticoagulation, a transesophageal echocardiogram may be performed to rule out left atrial thrombus. If this is not present, cardioversion may be attempted, with anticoagulation continued for 1 month if successful. Transthoracic echo-cardiography is inadequate to rule out left atrial thrombus. The patient is hemodynamically stable and has no indications for acute cardioversion. Dabigatran is not currently FDA approved for atrial flutter. Intravenous heparin should be started immediately if there are no contraindications, given the greater than 12-hour duration of symptoms.

37. The answer is B.

(Chap. 16) The ECG shows at least three different P-wave morphologies with three different PR intervals, which is the hallmark of multifocal atrial tachycardia. This is the signature tachycardia of patients with significant pulmonary disease and is commonly seen in patients with chronic obstructive pulmonary disease, as suggested by diffuse polyphonic expiratory wheezing and hyperinflation.

38. The answer is D.

(Chap. 16) The patient has classic symptoms for an AV nodal reentrant tachycardia. The so-called frog sign (prominent venous pulsations in the neck due to cannon A waves seen in AV dissociation) on physical examination is frequently present and suggests simultaneous atrial and ventricular contraction. First-line therapy for these reentrant narrow complex tachyarrhythmias is carotid sinus massage to increase vagal tone. Often this is all that is required to return the patient to sinus rhythm. If that is not successful, IV adenosine 6–12 mg may be attempted. If adenosine fails, intravenous beta blockers or calcium channel blockers may be used (diltiazem or verapamil). Finally, in hemodynamically compromised patients or those who have failed to respond to previous measures, DC cardioversion with 100–200 J is indicated.

39. The answer is D.

(Chap. 16) The patient has an accessory conduction pathway, as evidenced by the delta waves on his baseline ECG. He now presents with atrial fibrillation through the accessory pathway. The wide complex is not due to ventricular arrhythmia but rather the aberrant accessory conduction through the accessory pathway. In general, this reentrant tachycardia may be treated as all others, with the exception of avoiding digoxin and verapamil, both of which may cause deterioration to ventricular fibrillation. Digoxin is thought to shorten the refractory period of the accessory pathway and thus can precipitate degeneration to ventricular fibrillation. Verapamil is thought to cause systemic vasodilation, with a resultant increase in sympathetic tone, and thus may precipitate ventricular fibrillation as well.

40. The answer is A.

(Chap. 16) Atrial-ventricular dissociation is a classic finding in ventricular tachycardia. Physical examination may show jugular vein cannon A waves when the atria contracts against a closed tricuspid valve and the ECG will manifest this with atrial capture and/or fusion beats. Other findings on ECG of ventricular tachycardia include QRS duration greater than 140 milliseconds for right bundle branch pattern in V1 or greater than 160 milliseconds for left bundle morphology in lead V1, frontal plane axis of –90 to 180°, delayed activation during initial phase of the QRS complex, or bizarre QRS pattern that does not mimic typical right or left bundle branch block QRS complex patterns. An irregularly irregular rhythm with changing QRS complexes suggests atrial fibrillation with ventricular preexcitation. Carotid sinus massage, aimed at increasing vagal tone and slowing AV node conduction, is not effective at slowing ventricular tachycardia because the reentrant focus is below the AV node.

41 and 42. The answers are C and C, respectively.

(Chap. 16) The patient’s rhythm is torsade de pointes, with polymorphic ventricular tachycardia and QRS complexes with variations in amplitude and cycle length, giving the appearance of oscillation about an axis. Torsades de pointes are associated with a prolonged QT interval; thus, anything that is associated with a prolonged QT can potentially cause torsade. Most commonly, electrolyte disturbances such as hypokalemia and hypomagnesemia, phenothiazines, fluoroquinolones, antiarrhythmic drugs, tricyclic antidepressants, intracranial events, and bradyarrhythmias are associated with this malignant arrhythmia. Management, besides stabilization, which may require electrical cardioversion, consists of removing the offending agent. In addition, success in rhythm termination or prevention has been reported with the administration of magnesium as well as overdrive atrial or ventricular pacing, which will shorten the QT interval. Beta blockers are indicated for patients with congenital long QT syndrome, but are not indicated in this patient.

43. The answer is C.

(Chap. 16) There are three main mechanisms by which arrhythmias are initiated and maintained: automaticity, afterdepolarizations, and reentry. Automaticity, such as that seen with sinus tachycardia, atrial premature complexes, and some atrial tachycardias, is due to an increase in the slope of phase 4 of the action potential. The depolarization threshold is reached more quickly and repeatedly. Afterdepolarizations are associated with an increase in cellular calcium accumulation, leading to repeated myocardial depolarization during phase 3 (early) and phase 4 (delayed) of the action potential. Early afterdepolarizations may be related to the initiation of torsades de pointes. Delayed afterdepolarizations are responsible for arrhythmias related to digoxin toxicity and for catecholamine-induced ventricular tachycardia. Reentry is due to inhomogeneities in myocardial conduction and refractory periods. With reentry, conduction is blocked in one pathway, allowing slow conduction in the other. This allows for sufficient delay so that the blocked site has time for reentry and propagation of the tachycardia within the two pathways. Reentry appears to be the mechanism for most supraventricular and ventricular tachycardias.

44. The answer is C.

(Chap. 16) The mechanisms for atrial fibrillation initiation and maintenance are still debated; however, there are anatomic structures that play a role in both of these processes. Muscularized tissue at the orifices of the pulmonary vein inlets are the predominant anatomic drivers of atrial fibrillation, although metabolic disturbances (e.g., hyperthyroidism, inflammation, infection) are also very common. Radio-frequency ablation of the tissue in the area of the pulmonary vein inlets can terminate atrial fibrillation; however, recurrences are not uncommon and other anatomic drivers may be present. The left atrial appendage is an important site of thrombus formation in patients with atrial fibrillation. Any focus within the left or right atrium can be a focus of reentry of focal atrial tachycardia, including the mitral annulus or sinus venosus. Increased automaticity of the sinus node is the mechanism for sinus tachycardia.

45. The answer is B.

(Chap. 16) Symptoms of atrial fibrillation vary dramatically. The most common symptom is tachypalpitations; however, the hemodynamic effects account for symptoms of impaired left ventricular filling. In atrial fibrillation, there is not an effective atrial contraction to augment late-diastolic left ventricular filling. In patients with impaired ventricular diastolic function, this loss of effective atrial contraction causes impaired left ventricular filling, increased left atrial filling pressures, and pulmonary congestion. These hemodynamic effects are more common in the elderly and in patients with longstanding hypertension, hypertrophic cardiomyopathy, and obstructive aortic valve disease. The tachycardia of atrial fibrillation further compromises left ventricular filling and increases atrial filling pressures. Atrial fibrillation may occur with acute alcohol intoxication, with warming of hypothermic patients, and postoperatively after thoracic surgery. The magnitude of the hemodynamic effect and symptoms will be related to ventricular rate (a slower rate allows more time for left ventricular filling) and underlying cardiac function.

46. The answer is E.

(Chap. 17) The patient presents with evidence of heart failure by history, and physical examination confirms this diagnosis. Physical examination also shows exophthalmos and a fine tremor, which are suggestive of hyperthyroidism. Thyrotoxicosis, along with anemia, nutritional disorders, and systemic arteriovenous shunting, can all cause high-output heart failure. Although systolic and diastolic dysfunction are more common causes of heart failure, disorders associated with a high-output state are often reversible, and therefore a diagnosis should be pursued when clinical clues suggest this may be present.

47. The answer is C.

(Chap. 17) Circulating levels of natriuretic peptides may be a useful adjunctive tool in the diagnosis of heart failure, but they cannot replace clinical judgment. BNP or N-terminal BNP are most commonly used and are released from the failing heart, though their release is not specific to left or right heart failure; thus, elevations are commonly seen in cor pulmonale associated with pulmonary vascular disease as well as in patients with left heart failure. Additionally, there are a number of factors that may affect the level of BNP that is normally released from the failing heart. Age and renal dysfunction increase plasma BNP levels. Obesity is associated with falsely low BNP levels. Although BNP levels may normalize after therapy, serial monitoring of this peptide is not presently recommended as a guide for heart failure therapy.

48. The answer is E.

(Chap. 17) Several drugs have been shown to prevent disease progression in heart failure including ACE inhibitors, angiotensin receptor blockers, beta blockers, and aldosterone antagonists. ACE inhibition has been shown to improve symptoms and survival, reduce cardiac hypertrophy, and reduce hospitalizations. Its use is often complicated by cough related to kinin potentiation, which is an acceptable reason to switch to an angiotensin receptor blocker. Digoxin therapy has not been shown to improve survival, may be associated with dose toxicity, and in patients with stable disease who are not frequently hospitalized, can usually be withdrawn. Beta blocker therapy may occasionally be associated with worsening heart failure symptoms at the time of initiation, but this can usually be managed with increased diuretics. The benefits of beta blockers would far outweigh the nuisance of occasional extra diuretics in this patient. Aldosterone antagonists such as spironolactone and eplerenone are recommended for patients with EF less than 35% who are receiving standard therapy as above. There is no known benefit to one member of this class of drugs over another.

49. The answer is E.

(Chap. 17) Although there is a wealth of information on which drugs will improve symptoms and survival in heart failure with reduced ejection fraction, little is known about heart failure with preserved ejection fraction. In fact, there are no proven or approved pharmacologic therapies for patients with heart failure and preserved ejection fraction. Therapy should be aimed at treating the predisposing factors for development of this condition, i.e., treat systemic hypertension if present, reverse ischemia if appropriate, etc. Precipitating factors, such as dietary indiscretion in this patient, atrial fibrillation, or infection, may be addressed to improve symptoms. Sildenafil is currently only approved for therapy of pulmonary arterial hypertension and is not proven to be useful for pulmonary hypertension associated with heart failure with preserved ejection fraction.

50. The answer is C.

(Chap. 17) The New York Heart Association (NYHA) classification is a tool to define criteria that describe the functional ability and clinical manifestations of patients in heart failure. It is also used in patients with pulmonary hypertension. These criteria have been shown to have prognostic value with worsening survival as class increases. They are also useful to clinicians when reading studies to understand the entry and exclusion criteria of large clinical trials. Class I is used for patients with no limiting symptoms; class II for patients with slight or mild limitation; class III implies no symptoms at rest but dyspnea or angina or palpitations with little exertion—patients are moderately limited; class IV is used for severely limited patients in whom even minimal activity causes symptoms. Treatment guidelines also frequently base recommendations on these clinical stages. This patient has symptoms with mild exertion but is comfortable at rest; therefore, he is NYHA class III.

51. The answer is B.

(Chap. 17) Patients with severe congestive heart failure often exhibit Cheyne-Stokes breathing, defined as intercurrent short periods of hypoventilation and hyperventilation. The mechanism is thought to relate to the prolonged circulation time between the lungs and the respiratory control centers in the brain, leading to poor respiratory control of PaCO2. The degree of Cheyne-Stokes breathing is related to the severity of heart failure. This pattern of breathing is different from obstructive sleep apnea, which is notable for loud snoring, periods of apnea, and sudden waking. Patients are also often hypersomnolent during the day. While sleep apnea is managed with weight loss and overnight CPAP, Cheyne-Stokes breathing is difficult to address as it is often a sign of advanced systolic dysfunction and implies a poor prognosis. All efforts to further maximize heart failure management are indicated. A sleep study would demonstrate this pattern of breathing, but this history and clinical presentation is typical. There is no role for bronchodilators or an electroencephalogram.

52. The answer is B.

(Chap. 18) Coronary artery disease is a common late complication after cardiac transplantation and is thought to be due to a primary immunologic injury of the vascular endothelium, though it is influenced by nonimmunologic factors such as dyslipidemia, diabetes mellitus, and cytomegalovirus infection. Use of mycophenolate, mofetil, and the mammalian target of rapamycin sirolimus have been associated with a lower short-term incidence of coronary intimal thickening. Similarly, statin use has been shown to reduce the incidence of this complication. Because donors are generally young, the coronary artery disease after transplantation is not thought to be due to coronary lesions present pretransplantation.

53. The answer is E.

(Chap. 17) Ventricular assist device therapy can be used either as a “bridge” to transplantation in eligible candidates or as a final destination in patients with end-stage heart failure who are not transplant candidates. There are four FDA-approved devices, all of which share common complications including thromboembolism, cerebrovascular accident, device failure, and infection.

54. The answer is E.

(Chap. 19) Atrial septal defect (ASD) is a not uncommon simple congenital heart disease lesion that is often diagnosed in adults. Because of chronic left-to-right shunting of intracardiac blood, pulmonary arterial hypertension is a well-recognized common complication. With the development of pulmonary arterial hypertension, the potential for paradoxical embolization of either air or thrombotic material from the right atrium to the systemic circulation is increased. Similarly, with exertion in the context of pulmonary arterial hypertension and ASD, blood may shunt right to left, leading to systemic arterial oxygen desaturation. Atrial fibrillation or other supraventricular arrhythmias may occur, also as a result of atrial stretching with the lesion. While atherosclerosis and unstable angina may certainly occur in adults, is not a reported complication of ASD.

55. The answer is D.

(Chap. 19) The patient has secondary erythrocytosis due to Eisenmenger’s syndrome and chronic arterial hypoxemia. Her partially corrected left-to-right shunt resulted in chronic pulmonary circulation overflow and the subsequent development of pulmonary arterial hypertension. With a rise in pulmonary vascular pressure, the shunt reverses to become predominantly right to left, which causes systemic oxygen desaturation. Because hypoxemia is caused by shunt and not ventilation/perfusion mismatch (as in typical COPD), it is not responsive to oxygen therapy. Peripheral desaturation results in decreased oxygen delivery to the kidneys, increased erythropoietin secretion, and resultant erythrocytosis. Erythropoietin levels would be expected to be elevated in this case (in contrast to polycythemia vera rubra). Phlebotomy is only used for patients with symptomatic erythrocytosis; hyperviscosity symptoms, including neurologic symptoms such as transient ischemic attack; epistaxis or bleeding symptoms; or visual changes. Because iron depletion may worsen viscosity even at a lower hematocrit, it is considered as only a temporary therapy for management of erythrocytosis in Eisenmenger’s syndrome. This patient had no symptoms referable to erythrocytosis; therefore, expectant management is most appropriate.

56. The answer is D.

(Chap. 19) Routine antibiotic prophylaxis is indicated for bacteremic dental procedures or instrumentation through an infected site in most patients with operated congenital heart disease, particularly whenever foreign material is present. The one exception is patches that don’t have a post-placement high-grade leak, where prophylaxis is only required for 6 months until endothelialization.

57. The answer is E.

(Chap. 19) The patient presents with dextrocardia on his chest radiograph and situs inversus, or complete mirror image situs inversus on examination. When dextrocardia occurs in isolation without situs inversus, multiple cardiac abnormalities are frequently present. Alternatively, when dextrocardia occurs with situs inversus, other cardiac defects are unlikely. Kartagener’s syndrome with mucociliary dysfunction may underlie situs inversus, but it is associated with sinusitis and chronic bronchitis, which this patient did not have.

58 and 59. The answers are C and A, respectively.

(Chap. 19) This patient has a coarctation of the aorta presenting with marked hypertension proximal to the lesion. The narrowing most commonly occurs distal to the origin of the left subclavian artery, explaining the equal pressure in the arms and reduced pressure in the legs. Coarctations account for approximately 7% of congenital cardiac abnormalities, occur more frequently (2×) in men than in women, and are associated with gonadal dysgenesis and bicuspid aortic valves. Adults will present with hypertension, manifestations of hypertension in the upper body (headache, epistaxis), or leg claudication. Physical examination reveals diminished and/or delayed lower extremity pulses, enlarged collateral vessels in the upper body, or reduced development of the lower extremities. Cardiac examination may reveal findings consistent with left ventricular (LV) hypertrophy. There may be no murmur, a mid-systolic murmur over the anterior chest and back, or an aortic murmur with a bicuspid valve. Transthoracic (suprasternal/parasternal) or transesophageal echocardiography, contrast CT or MRI of the thorax, or cardiac catheterization can be diagnostic. MRI of the head would not be useful diagnostically. The clinical picture is not consistent with renal artery stenosis, pheochromocytoma, carcinoid, or Cushing’s syndrome.

60. The answer is E.

(Chap. 20) Mitral stenosis is one of the leading causes of pulmonary hypertension worldwide, particularly in developing countries where the treatment of streptococcal disease is less available. The primary determinants of pulmonary artery pressure are left atrial pressure, pulmonary vascular resistance, and flow. Mitral stenosis may restrict flow from the left atrium to the left ventricle, and thus is associated with left atrial hypertension and passive pulmonary hypertension (due to back pressure). Additionally, the pulmonary vascular bed may actively vasoconstrict in response to left atrial hypertension. Additional contributors to pulmonary hypertension in mitral stenosis include interstitial edema in the walls of small pulmonary vessels and, in end-stage disease, obliterative changes in the pulmonary vascular bed as may be seen in some forms of pulmonary arterial hypertension. Pulmonary hypertension related to mitral stenosis is generally reversible with correction of the valvular lesion.

61 and 62. The answers are A and E, respectively.

(Chap. 20) The patient presents with a relatively stable ST elevation myocardial infarction. He likely has extensive necrosis given the duration of symptoms and ECG findings, and thus is at risk for complication of myocardial infarction. In this case, his acute dyspnea, worsening oxygenation, and asymmetric edema on chest radiograph all point to acute mitral regurgitation from papillary muscle rupture. An allergic reaction to a medication should not cause severe hypoxemia. It may cause rather mild reversible hypoxemia, and should not cause an abnormal chest radiograph. The classic finding of acute mitral regurgitation is a relatively loud systolic murmur heard best at the apex and radiating to the axilla. The murmur is described as having a “cooing” or “seagull-like” quality. A fourth heart sound is also common. Management of acute mitral regurgitation includes afterload and preload reduction, if possible, often with intravenous nitroprusside. If patients are unable to tolerate medical interventions to achieve this because of systemic hypotension, as in this patient, an intraaortic balloon pump is indicated. Albuterol and methylprednisolone are indicated for acute bronchospasm due to primary airways disease, but would not be helpful for the management of cardiogenic shock.

63. The answer is C.

(Chap. 20) The patient has classic physical examination findings for mitral valve prolapse with a mid-systolic click that may or may not be associated with a systolic murmur. Mitral valve prolapse is generally thought to be a benign lesion, with most patients never developing symptoms during their lifetimes. While many patients with heritable connective tissue disorders such as Marfan’s syndrome have mitral valve prolapse, in the majority of cases, a cause is not identified. Mitral valve prolapse may be seen on echocardiography by systolic displacement of the mitral valve leaflets by at least 2 mm into the left atrium. Doppler imaging may also be helpful to define the condition. Because the lesion is generally benign, endocarditis prophylaxis is generally not indicated unless the patient has a prior history of endocarditis. Although some patients develop atrial arrhythmias in conjunction with mitral valve prolapse, prophylactic antiplatelet agents or warfarin are not recommended, as most patients do not have complications.

64. The answer is B.

(Chap. 20) The patient has aortic stenosis that presented late in life. While bicuspid aortic valve underlies nearly half of all aortic stenosis cases, this lesion typically presents earlier in life, and only 40% of patients greater than 70 years old with aortic stenosis who undergo surgery have a bicuspid valve. Rheumatic heart disease may cause aortic stenosis, but almost invariably mitral stenosis is also present. Underlying connective tissue disease is not known to be associated. Modern research on the development of aortic stenosis has shown that several traditional atherosclerotic risk factors are present such as diabetes mellitus, smoking, chronic kidney disease, and the metabolic syndrome. Polymorphisms of the vitamin D receptor have also been demonstrated in patients with symptomatic aortic stenosis.

65. The answer is C.

(Chap. 20) Exertional syncope is a late finding in aortic stenosis (AS) and portends a poor prognosis. Patients with this symptom or with angina pectoris have an average time to death of 3 years. Patients with dyspnea have 2 years, and patients with heart failure have an average time to death of 1.5–2 years. Because of these data, patients with severe AS and symptoms should be strongly considered for surgical therapy.

66. The answer is A.

(Chap. 20) Patients with severe aortic regurgitation will have a “water-hammer” pulse that collapses suddenly as arterial pressure rapidly falls during late systole and diastole, a so-called Corrigan’s pulse. Capillary pulsations seen in the nail bed in severe aortic regurgitation are named Quincke’s pulse. Traube’s sign, or a pistol shot sound, may be heard over the femoral arteries and Duroziez’s sign, with a to-and-fro murmur over the femoral artery, have also been described. Pulsus parvus et tardus is found in severe aortic stenosis. Pulsus bigeminus occurs when there is a shorter interval after a normal beat with a following low volume pulse, often with a premature ventricular beat. Pulsus paradoxus has been described with pericardial tamponade or severe obstructive lung disease. Pulsus alternans is alternating large and small volume pulses seen in severe heart failure.

67 and 68. The answers are C and C, respectively.

(Chap. 20) The patient presents with heart failure during her second trimester from a region with high rates of rheumatic fever. She is therefore at risk for rheumatic mitral stenosis, which often presents during the second trimester of pregnancy as the cardiac output must rise to accommodate the fetus and intravascular volume expands substantially. The stenotic valve cannot accommodate the increased flow demands of pregnancy, and congestive heart failure ensues with secondary pulmonary venous hypertension. The patient has evidence of heart failure on examination with pulmonary hypertension. Her diastolic rumble is characteristic of mitral stenosis. Finally, hemoptysis is not an infrequent finding in severe mitral stenosis and may be due to the rupture of pulmonary-bronchial venous connections secondary to pulmonary venous hypertension. Occasionally, pink frothy sputum can be found in patients with frank alveolar hemorrhage related to elevated pulmonary capillary pressure. Mitral stenosis is readily demonstrated by echocardiography. While right heart catheterization may demonstrate pulmonary hypertension and an elevated pulmonary capillary wedge pressure, the etiology of these findings will remain unknown without imaging of the left heart. Short-term management of mitral stenosis with heart failure should include diuretics. As the patient does not have left ventricular failure, ACE inhibition and digoxin are not likely to alleviate her symptoms. Occasionally, beta blockade may improve symptoms, particularly in patients with symptomatic atrial arrhythmias. Anticoagulation is not indicated in mitral stenosis alone unless atrial arrhythmias or pulmonary embolism is present. As infection does not underlie the patient’s hemoptysis, further antibiotics will not be helpful.

69. The answer is D.

(Chap. 20) Indications for surgical repair of mitral regurgitation are dependent on left-ventricular function, ventricular size, and the presence of sequelae of chronic mitral regurgitation. The experience of the surgeon and the likelihood of successful mitral valve repair are also important considerations. The management strategy for chronic severe mitral regurgitation depends on the presence of symptoms, left-ventricular function, left-ventricular dimensions, and the presence of complicating factors such as pulmonary hypertension and atrial fibrillation. With very depressed left-ventricular function (<30% or endsystolic dimension >55 mm), the risk of surgery increases, left-ventricular recovery is often incomplete, and long-term survival is reduced. However, since medical therapy offers little for these patients, surgical repair should be considered if there is a high likelihood of success (>90%). When ejection fraction is between 30% and 60%, and end-systolic dimension rises above 40 mm, surgical repair is indicated even in the absence of symptoms, owing to the excellent long-term results achieved in this group. Waiting for worsening left-ventricular function leads to irreversible left-ventricular remodeling. Pulmonary hypertension and atrial fibrillation are important to consider as markers for worsening regurgitation. For asymptomatic patients with normal left-ventricular function and dimensions, the presence of new pulmonary hypertension or atrial fibrillation in patients with normal ejection fraction and end-systolic dimensions are class IIa indications for mitral valve repair.

70. The answer is F.

(Chap. 20) Tricuspid regurgitation is most commonly caused by dilation of the tricuspid annulus due to right-ventricular enlargement of any cause. Any cause of left-ventricular failure that results in right-ventricular failure may lead to tricuspid regurgitation. Congenital heart diseases or pulmonary arterial hypertension leading to right-ventricular failure will dilate the tricuspid annulus. Inferior wall infarction may involve the right ventricle. Rheumatic heart disease may involve the tricuspid valve, although less commonly than the mitral valve. Infective endocarditis, particularly in IV drug users, will infect the tricuspid valve, causing vegetations and regurgitation. Other causes of tricuspid regurgitation include carcinoid heart disease, endomyocardial fibrosis, congenital defects of the atrioventricular canal, and right-ventricular pacemakers.

71. The answer is A.

(Chap. 20) Bioprosthetic valves are made from human, porcine, or bovine tissue. The major advantage of a bioprosthetic valve is the low incidence of thromboembolic phenomena, particularly 3 months after implantation. Although in the immediate postoperative period some anticoagulation may occur, after 3 months there is no further need for anticoagulation or monitoring. The downside is the natural history and longevity of the bioprosthetic valve. Bioprosthetic valves tend to degenerate mechanically. Approximately 50% will need replacement at 15 years. Therefore, these valves are useful in patients with contraindications to anticoagulation, such as elderly patients with comorbidities and younger patients who desire to become pregnant. Elderly people may also be spared the need for repeat surgery, as their life span may be shorter than the natural history of the bioprosthesis. Mechanical valves offer superior durability. Hemodynamic parameters are improved with double-disk valves compared with single-disk or ball-and-chain valves. However, thrombogenicity is high and chronic anticoagulation is mandatory. Younger patients with no contraindications to anticoagulation may be better served by mechanical valve replacement.

72. The answer is E.

(Chap. 21) Many infectious etiologies have been associated with the development of inflammatory myocarditis including viral agents (coxsackie, adenovirus, HIV, hepatitis C) and parasitic agents, with Chagas disease or T. cruzi being most prominent, but also toxoplasmosis. Additionally, bacterial etiologies like diphtheria, spirochetal disease like Borrelia burgdorferi, rickettsial disease, and fungal infections have been associated.

73. The answer is C.

(Chap. 21) Peripartum cardiomyopathy is a rare complication of pregnancy and can occur during the last trimester or within the first 6 months postpartum. Risk factors include advanced age, increased parity, twin pregnancy, malnutrition, use of tocolytic therapy for premature labor, and preeclampsia.

74. The answer is A.

(Chap. 21) Beriberi heart disease is a dilated cardiomyopathy due to thiamine deficiency. While uncommon in developed countries, this condition still occurs in patients who derive most of their calories from alcohol and has been reported in teenagers who eat only highly processed foods. This condition involves systemic vasodilation with a very high cardiac output in its early stages. In advanced disease, a low-output state can occur. Thiamine repletion can lead to a complete recovery. Patient A has evidence of heart failure with systemic vasodilation and elevated cardiac output, as would be found in beriberi. Alternatively, patient B has normal hemodynamics. Patient C has evidence of low-output heart failure with systemic vasoconstriction. Patient D has elevated pulmonary arterial pressures with right heart failure in conjunction with normal pulmonary capillary wedge pressure, consistent with primary pulmonary vascular disease, e.g., pulmonary arterial hypertension. Patient E has low right heart filling pressures, with somewhat low cardiac output and elevated systemic vascular resistance, as might be found in hypovolemic shock.

75. The answer is E.

(Chap. 21) Hypertrophic cardiomyopathy usually presents between age 20 and 40 years, with the most common symptom being dyspnea. Many patients are, however, asymptomatic and the only clue to the presence of this potentially deadly disease is physical examination. Physical examination will show a harsh systolic murmur heard best at the left lower sternal border arising from both the outflow tract turbulence during ventricular ejection and the often concomitant mitral regurgitation. Maneuvers that decrease ventricular volume such as Valsalva or moving from squatting to standing will enhance the murmur. Conversely, maneuvers that increase left ventricular volume will decrease the murmur’s intensity. These include hand grip and squatting. Having the patient lie with the left side down and leaning forward may make the friction rub of pericarditis more audible.

76. The answer is E.

(Chap. 21) A common diagnostic dilemma is differentiating constrictive pericarditis from a restrictive cardiomyopathy. Elevated jugular venous pressure is almost universally present in both. Kussmaul’s sign (increase or no change in jugular venous pressure with inspiration) can be seen in both conditions. Other signs of heart failure do not reliably distinguish the two conditions. In restrictive cardiomyopathy, the apical impulse is usually easier to palpate than in constrictive pericarditis, and mitral regurgitation is more common. These clinical signs, however, are not reliable to differentiate the two entities. In conjunction with clinical information and additional imaging studies of the left ventricle and pericardium, certain pathognomic findings increase diagnostic certainty. A thickened or calcified pericardium increases the likelihood of constrictive pericarditis. Conduction abnormalities are more common in infiltrating diseases of the myocardium. In constrictive pericarditis, measurements of diastolic pressures will show equilibrium between the ventricles, while unequal pressures and/or isolated elevated left ventricular pressures are more consistent with restrictive cardiomyopathy. The classic “square root sign” during right heart catheterization (deep, sharp drop in right ventricular pressure in early diastole, followed by a plateau during which there is no further increase in right ventricular pressure) can be seen in both restrictive cardiomyopathy and constrictive pericarditis. The presence of a paraprotein abnormality (MGUS, myeloma, amyloid) makes restrictive cardiomyopathy more common.

77. The answer is D.

(Chap. 21) Cardiac involvement is common in many of the neuromuscular diseases. The ECG pattern of Duchenne’s muscular dystrophy is unique and consists of tall R waves in the right precordial leads with an R/S ratio greater than 1.0, often with deep Q waves in the limb and precordial leads. These patients often have a variety of supraventricular and ventricular arrhythmias, and are at risk for sudden death due to the intrinsic cardiomyopathy as well as the low ejection fraction. Implantable cardioverter defibrillators should be considered in the appropriate patient. Global left ventricular dysfunction is a common finding in dilated cardiomyopathies, whereas focal wall motion abnormalities and angina are more common if there is ischemic myocardium. This patient is at risk for venous thromboembolism; however, chronic thromboembolism would not account for the severity of the left heart failure and would present with findings consistent with pulmonary hypertension. Amyotrophic lateral sclerosis is a disease of motor neurons and does not involve the heart. This patient would be young for that diagnosis. An advanced atrial septal defect would present with cyanosis and heart failure (Eisenmenger’s physiology).

78. The answer is D.

(Chap. 22) The patient has a classic presentation for acute pericarditis with constant or pleuritic chest pain, exacerbated by lying flat and alleviated by sitting forward. Serum biomarkers may show mild evidence of myocardial injury from myocardial inflammation, but are generally not substantially elevated. Friction rub is frequently present, has three components, and is best heard while the patient is upright and leaning forward. In the acute stages, ECG classically shows ST-segment elevation with upward concavity in two or three standard limb leads and V2 through V6 with reciprocal changes in aVR. Convex curvature is more commonly found in acute myocardial infarction. PR depression may be found. After several days, the ST changes resolve and T waves become inverted. After weeks to months, the ECG returns to normal.

79. The answer is B.

(Chap. 22) Pulsus paradoxus is an exaggeration of the normal phenomenon in which systolic blood pressure declines 10 mmHg or less with inspiration. Pulsus paradoxus is typically seen in patients with pericardial tamponade and in patients with severe obstructive lung disease (COPD, asthma). In pulsus paradoxus due to pericardial tamponade, the inspiratory systolic blood pressure decline is greater due to the tight incompressible pericardial sac. The right ventricle distends with inspiration, compressing the left ventricle and resulting in decreased systolic pulse pressure in the systemic circulation. In severe obstructive lung disease, the inspiratory decline of systolic blood pressure may be due to the markedly negative pleural pressure either causing left ventricular compression (due to increased RV venous return) or increased LV impedance to ejection (increased afterload).

80. The answer is C.

(Chap. 22) Beck’s triad can be used to alert clinicians to the potential presence of cardiac tamponade. The principal features are hypotension, muffled or absent heart sounds, and elevated neck veins, often with prominent x-descent and absent y-descent. These are due to the failure of ventricular filling and limited cardiac output. Kussmaul’s sign is seen in restrictive cardiomyopathy and pericardial constriction, not tamponade. Friction rub may be seen in any condition associated with pericardial inflammation.

81. The answer is D.

(Chap. 22) This patient’s presentation and physical examination are most consistent with the diagnosis of constrictive pericarditis. The most common cause of constrictive pericarditis worldwide is tuberculosis, but given the low incidence of tuberculosis in the United States, constrictive pericarditis is a rare condition in this country. With the increasing ability to cure Hodgkin’s disease with mediastinal irradiation, many cases of constrictive pericarditis in the United States involve patients who received curative radiation therapy 10–20 years prior. These patients are also at risk for premature coronary artery disease. Risks for these complications include dose of radiation and radiation windows that include the heart. Other rare causes of constrictive pericarditis are recurrent acute pericarditis, hemorrhagic pericarditis, prior cardiac surgery, mediastinal irradiation, chronic infection, and neoplastic disease. Physiologically, constrictive pericarditis is characterized by the inability of the ventricles to fill because of the noncompliant pericardium. In early diastole, the ventricles fill rapidly, but filling stops abruptly when the elastic limit of the pericardium is reached. Clinically, patients present with generalized malaise, cachexia, and anasarca. Exertional dyspnea is common, and orthopnea is generally mild. Ascites and hepatomegaly occur because of increased venous pressure. In rare cases, cirrhosis may develop from chronic congestive hepatopathy. The jugular venous pressure is elevated, and the neck veins fail to collapse on inspiration (Kussmaul’s sign). Heart sounds may be muffled. A pericardial knock is frequently heard. This is a third heart sound that occurs 0.09–0.12 seconds after aortic valve closure at the cardiac apex. Right heart catheterization would show the “square root sign” characterized by an abrupt y-descent followed by a gradual rise in ventricular pressure. This finding, however, is not pathognomonic of constrictive pericarditis and can be seen in restrictive cardiomyopathy of any cause. Echocardiogram shows a thickened pericardium, dilatation of the inferior vena cava and hepatic veins, and an abrupt cessation of ventricular filling in early diastole. Pericardial resection is the only definitive treatment of constrictive pericarditis. Diuresis and sodium restriction are useful in managing volume status preoperatively, and paracentesis may be necessary. Operative mortality ranges from 5–10%. Underlying cardiac function is normal; thus, cardiac transplantation is not indicated. Pericardiocentesis is indicated for the diagnostic removal of pericardial fluid and cardiac tamponade, which is not present on the patient’s echocardiogram. Mitral valve stenosis may present similarly with anasarca, congestive hepatic failure, and ascites. However, pulmonary edema and pleural effusions are also common. Examination would be expected to demonstrate a diastolic murmur, and echocardiogram should show a normal pericardium and a thickened immobile mitral valve. Mitral valve replacement would be indicated if mitral stenosis were the cause of the patient’s symptoms.

82. The answer is C.

(Chap. 23) Blunt, nonpenetrating trauma such as that described here can result in commotio cordis, which occurs when the trauma impacts the heart during the susceptible phase of repolarization just before the peak of the T wave and results in ventricular fibrillation. This syndrome is most common in young athletes who are playing hockey, football, baseball, or lacrosse, for example. Treatment is prompt defibrillation. While aortic rupture, myocardial rupture with cardiac tamponade, and tension pneumothorax may occur with chest wall trauma, their presentation should be less immediate after the trauma. Hypertrophic cardiomyopathy may present with sudden cardiac death, as in this case, but the preceding chest trauma makes commotio cordis more likely.

83. The answer is E.

(Chap. 25) The etiologic agents of infective endocarditis vary by host (see Figure 25-1). Community-acquired native valve endocarditis remains an important clinical problem, particularly in elderly people. In those patients, streptococci (Viridans spp., S. gallolyticus, other nongroup A and other group streptococci, and Abiotrophia spp.) account for approximately 40% of cases. Staphylococcus aureus (30%) is next most common. Enterococci, HACEK group, coagulase-negative, and culture-negative cases each account for less than 10% of community-acquired native valve cases. In health care–associated, injection drug use–associated, and greater than 12-month-old prosthetic valve endocarditis, S. aureus is most common. Coagulasenegative staphylococcus is the most common organism in prosthetic valve endocarditis less than 12 months. Enterococci cause endocarditis in approximately 10% to 15% of cases in health care–associated, 2- to 12-month prosthetic valve, and injection drug use cases. Culture-negative endocarditis accounts for 5% to 10% of cases in all of the aforementioned clinical scenarios.

84. The answer is B.

(Chap. 25) The Duke criteria for diagnosis of infective endocarditis are a set of major and minor clinical, laboratory, and echocardiographic criteria that are highly sensitive and specific. The presence of two major criteria, one major criterion and three minor criteria, or five minor criteria allows a clinical diagnosis of definite endocarditis (see Table 25-3). Evidence of echocardiographic involvement as evidenced by an oscillating mass (vegetation) on a valve, supporting structure, or implanted material; an intracardiac abscess or partial dehiscence of a prosthetic valve; or a new valvular regurgitation are major criteria in the Duke classification. An increase or change in preexisting murmur by clinical examination is not sufficient. Transthoracic echocardiography is specific for infective endocarditis but only finds vegetations in about 65% of patients with definite endocarditis. It is not adequate for evaluation of prosthetic valves or for intracardiac complications. Transesophageal echocardiography is more sensitive, detecting abnormalities in more than 90% of cases of definite endocarditis.

85. The answer is C.

(Chap. 25) The recommendations for prophylaxis to prevent infective endocarditis have undergone change recently with a change to recommending it for fewer patients. The most recent American Heart Association guidelines (Circulation 116:1736, 2007) reverse many of the former recommendations based on indirect evidence suggesting that benefit is minimal and is not supported by cost-benefit or cost-effectiveness studies. Current recommendations advise prophylactic antibiotics only for those at highest risk for severe morbidity or mortality from endocarditis undergoing manipulation of gingival tissue or periapical region of the teeth, perforation of the oral mucosa, or a procedure on an infected site. Prophylaxis is not advised for routine gastrointestinal or genitourinary procedures. High-risk patients include those with prior endocarditis, prosthetic heart valves, unrepaired cyanotic congenital heart disease lesions, recently (<6 months) repaired congenital heart lesions, incompletely repaired congenital heart disease lesions, and valvulopathy after cardiac transplant. The British Society for Antimicrobial Chemotherapy does recommend prophylaxis for at-risk patients undergoing selected gastrointestinal or genitourinary procedures; however, the National Institute for Health and Clinical Excellence in the United Kingdom advised discontinuation of the practice (http://www.nice.org.uk/guidance/cg64).

86. The answer is A.

(Chap. 25) This patient has culture-negative endocarditis, a rare entity defined as clinical evidence of infectious endocarditis in the absence of positive blood cultures. In this case, evidence for subacute bacterial endocarditis includes valvular regurgitation; an aortic valve vegetation; and embolic phenomena on the extremities, spleen, and kidneys. A common reason for negative blood cultures is prior antibiotics. In the absence of this, the two most common pathogens (both of which are technically difficult to isolate in blood culture bottles) are Q fever, Coxiella burnetii (typically associated with close contact with livestock), and Bartonella spp. In this case, the patient’s homelessness and body louse infestation are clues for Bartonella quintana infection. Diagnosis is made by blood culture about 25% of the time. Otherwise, direct polymerase chain reaction of valvular tissue, if available, or acute and convalescent serologies are diagnostic options. Empirical therapy for culture-negative endocarditis usually includes ceftriaxone and gentamicin with or without doxycycline. For confirmed Bartonella endocarditis, optimal therapy is gentamicin plus doxycycline. EBV and HIV do not cause endocarditis. A peripheral blood smear would not be diagnostic.

87. The answer is D.

(Chap. 25) Although any valvular vegetation can embolize, vegetations located on the mitral valve and vegetations larger than 10 mm are greatest risk of embolizing. Of the answer choices, C, D, and E are large enough to increase the risk of embolization. However, only choice D demonstrates the risks of both size and location. Hematogenously seeded infection from an embolized vegetation may involve any organ but particularly affects those organs with the highest blood flow. They are seen in up to 50% of patients with endocarditis. Tricuspid lesions lead to pulmonary septic emboli, which are common in injection drug users. Mitral and aortic lesions can lead to embolic infections in the skin, spleen, kidneys, meninges, and skeletal system. A dreaded neurologic complication is mycotic aneurysm, focal dilations of arteries at points in the arterial wall that have been weakened by infection in the vasa vasorum or septic emboli, leading to hemorrhage.

88. The answer is A.

(Chap. 25) Patients with infective endocarditis on antibiotic therapy can be expected to demonstrate clinical improvement within 5 to 7 days. Blood cultures frequently remain positive for 3 to 5 days for Staphylococcus aureustreated with β-lactam antibiotics and 7 to 9 days with vancomycin. Neither rifampin nor gentamicin has been shown to provide clinical benefit in the scenario described in this question. Vancomycin peak and trough levels have not been shown to improve drug efficacy in infective endocarditis. It is too early in therapy to consider this case representative of vancomycin failure. The efficacy of daptomycin or linezolid because an alternative to vancomycin for left-sided MRSA endocarditis has not been established.

89. The answer is D.

(Chap. 26) Acute rheumatic fever (ARF) is almost universally due to group A streptococcal disease at the present time, though virtually all streptococcal disease may be capable of precipitating rheumatic fever. Although skin infections may be associated with rheumatic fever, far and away the most common presentation is with preceding pharyngitis. There is a latent period of approximately 3 weeks from an episode of sore throat to presentation of ARF. The most common manifestations are fever and polyarthritis, with polyarthritis being present in 60–75% of cases. Carditis may also be present, though somewhat less frequently in 50–60% of cases. Chorea and indolent carditis may have a subacute presentation. Chorea is present in 2–30% of affected individuals, while erythema marginatum and subcutaneous nodules are rare. Sixty percent of patients with ARF progress to rheumatic heart disease, with the endocardium, pericardium, and myocardium all potentially involved. All patients with ARF should receive antibiotics sufficient to treat the precipitating group A streptococcal infection.

90. The answer is D.

(Chap. 26) This patient has a history very suggestive of recurrent bouts of ARF with evidence of mitral regurgitation, mitral stenosis, and aortic regurgitation on physical examination. This and the presence of atrial fibrillation imply severe rheumatic heart disease. Risk factors for this condition include poverty and crowded living conditions. As a result, ARF is considerably more common in the developing world. Daily aspirin is the treatment of choice for the migratory large-joint arthritis and fever that are common manifestations of ARF. Practitioners sometimes use steroids during acute bouts of carditis to quell inflammation, though this remains a controversial practice and has no role between flares of ARF. Secondary prophylaxis with either daily oral penicillin or, preferably, monthly IM injections is considered the best method to prevent further episodes of ARF, and therefore prevent further valvular damage. Primary prophylaxis with penicillin on an as-needed basis is equally effective for preventing further bouts of carditis. However, most episodes of sore throat are too minor for patients to present to a physician. Therefore, secondary prophylaxis is considered preferable in patients who already have severe valvular disease. Doxycycline is not a first-line agent for group A Streptococcus, the pathogen that incites ARF.

91. The answer is A.

(Chap. 28) Cardiogenic shock (CS) is characterized by systemic hypoperfusion caused by severe depression of the cardiac index (<2.2 L/min/m2) and sustained systolic arterial hypotension (<90 mmHg) despite an elevated filling pressure (pulmonary capillary wedge pressure >18 mmHg). It is associated with in-hospital mortality rates above 50%. Acute myocardial infarction (MI) with left ventricular dysfunction is the most common cause of cardiogenic shock. Other complications of acute MI such as mitral regurgitation or free wall rupture are far less common. CS is the leading cause of death of patients hospitalized with MI. Early reperfusion therapy for acute MI decreases the incidence of CS. Shock typically is associated with ST-segment elevation MI and is less common with non–ST-segment elevation MI. In patients with acute MI, older age, female sex, prior MI, diabetes, and anterior MI location are all associated with an increased risk of CS. Shock associated with a first inferior MI should prompt a search for a mechanical cause. Reinfarction soon after MI increases the risk of CS. Two-thirds of patients with CS have flow-limiting stenoses in all three major coronary arteries, and 20% have stenosis of the left main coronary artery. CS may rarely occur in the absence of significant stenosis, as seen in LV apical ballooning/Takotsubo’s cardiomyopathy.

92. The answer is E.

(Chap. 28) In patients with acute myocardial infarction and cardiogenic shock, percutaneous coronary intervention can improve mortality rate and outcomes. Stabilizing the patient in cardiogenic shock is an important first maneuver. Initial therapy is aimed at maintaining adequate systemic and coronary perfusion by raising systemic blood pressure with vasopressors and adjusting volume status to a level that ensures optimum left ventricular filling pressure. Decreased diastolic blood pressure is detrimental because it reduces coronary blood flow. However, vasopressor and inotropic agents have the potential to exacerbate the ischemic process by raising myocardial oxygen consumption, increasing heart rate, or increasing left ventricular afterload. Norepinephrine is associated with fewer adverse events, including arrhythmias, compared with dopamine. Dobutamine has greater inotropic than chronotropic action but may cause a reduction in blood pressure due to vasodilation. Aortic counterpulsation with an intraaortic balloon pump (IABP) is helpful in rapidly stabilizing patients because it is capable of augmenting both arterial diastolic pressure and cardiac output. The balloon is automatically inflated during early diastole, augmenting coronary blood flow, and it collapses in early systole, reducing the left ventricular afterload. IABP improves hemodynamic status temporarily in most patients with cardiogenic shock. In contrast to vasopressors and inotropic agents, myocardial O2 consumption is reduced, ameliorating ischemia. IABP is contraindicated if aortic regurgitation is present or aortic dissection is suspected.

93. The answer is E.

(Chap. 29) The most common electrical mechanism for cardiac arrest is ventricular fibrillation, which is responsible for 50–80% of cardiac arrests. Severe persistent bradyarrhythmias, asystole, and pulseless electrical activity (PEA: organized electrical activity, unusually slow, without mechanical response, formerly called electromechanical dissociation) cause another 20–30%. Pulseless sustained ventricular tachycardia (a rapid arrhythmia distinct from PEA) is a less common mechanism. Acute low cardiac output states, having a precipitous onset also may present clinically as a cardiac arrest. These hemodynamic causes include massive acute pulmonary emboli, internal blood loss from a ruptured aortic aneurysm, intense anaphylaxis, and cardiac rupture with tamponade after myocardial infarction. Sudden deaths from these causes are not typically included in the category of sudden cardiac death.

94. The answer is A.

(Chap. 29) The probability of achieving successful resuscitation from cardiac arrest is related to the interval from onset of loss of circulation to institution of resuscitative efforts, the setting in which the event occurs, the mechanism (ventricular fibrillation, ventricular tachycardia, PEA, asystole), and the clinical status of the patient before the cardiac arrest. Return of circulation and survival rates as a result of defibrillation decrease almost linearly from the first minute to 10 minutes. After 5 minutes, survival rates are no better than 25–30% in out-of-hospital settings. Settings in which it is possible to institute prompt cardiopulmonary resuscitation (CPR) followed by prompt defibrillation provide a better chance of a successful outcome. However, the outcome in intensive care units (ICUs) and other in-hospital environments is heavily influenced by the patient’s preceding clinical status. The immediate outcome is good for cardiac arrest occurring in ICUs in the presence of an acute cardiac event or transient metabolic disturbance, but survival among patients with far-advanced chronic cardiac disease or advanced noncardiac diseases (e.g., renal failure, pneumonia, sepsis, diabetes, cancer) is low and not much better in the in-hospital than in the out-of-hospital setting. Survival from unexpected cardiac arrest in unmonitored areas in a hospital is not much better than that it is for witnessed out-of-hospital arrests. Because implementation of community response systems, survival from out-of-hospital cardiac arrest has improved, although it still remains low under most circumstances. Survival probabilities in public sites exceed those in the home environment. This may be because many patients with at home cardiac arrest have severe underlying cardiac disease. The success rate for initial resuscitation and survival to hospital discharge after an out-of-hospital cardiac arrest depends heavily on the mechanism of the event. Most cardiac arrests that are caused by ventricular fibrillation (VF) begin with a run of nonsustained or sustained ventricular tachycardia (VT), which then degenerates into VF. When the mechanism is pulseless VT, the outcome is best, VF is the next most successful, and asystole and PEA generate dismal outcome statistics. Advanced age also adversely influences the chances of successful resuscitation as well as outcomes after resuscitation.

95. The answer is D.

(Chap. 31) Mutation of the LDL receptor results in hypercholesterolemia. This mutation may be homozygous or heterozygous and occurs in approximately 1/500 people in its heterozygous form. Homozygous disease is more severe, with the development of symptomatic coronary atherosclerosis in childhood, while heterozygous patients have hypercholesterolemia from birth, and disease recognition is usually not until adulthood when patients are found to have tendon xanthomas or coronary artery disease. In patients with heterozygous disease, there is generally a family history on at least one side of the family. In familial hypercholesterolemia, there is an elevation of LDL-C between 200 and 400 mg/dL without alterations in chylomicrons or VLDL. Familial defective apoB-100 has a similar presentation but is less common (1/1000). Autosomal dominant history may be present in this family to suggest autosomal dominant hypercholesterolemia; however, this condition is quite rare (<1/1,000,000) and therefore much less likely. Familial hepatic lipase deficiency and lipoprotein lipase deficiency are associated with increased chylomicrons, not LDL-C, and present with eruptive xanthomas, hepatosplenomegaly, and pancreatitis. These conditions occur rarely (<1/1,000,000).

96. The answer is B.

(Chap. 31) There are many secondary forms of elevated LDL that warrant consideration in a patient found to have abnormal LDL. These include hypothyroidism, nephritic syndrome, cholestasis, acute intermittent porphyria, anorexia nervosa, hepatoma, and drugs such as thiazides, cyclosporine, and Tegretol. Cirrhosis is associated with reduced LDL because of inadequate production. Malabsorption, malnutrition, Gaucher’s disease, chronic infectious disease, hyperthyroidism, and niacin toxicity are all similarly associated with reduced LDL.

97. The answer is D.

(Chap. 31) This patient has signs and symptoms of familial hypercholesterolemia (FH) with elevated plasma LDL, normal triglycerides, tendon xanthomas, and premature coronary artery disease. FH is an autosomal codominant lipoprotein disorder that is the most common of these syndromes caused by a single gene disorder. It has a higher prevalence in Afrikaners, Christian Lebanese, and French Canadians. There is no definitive diagnostic test for FH. It may be diagnosed with a skin biopsy that shows reduced LDL receptor activity in cultured fibroblasts (although there is considerable overlap with normals). FH is predominantly a clinical diagnosis, although molecular diagnostics are being developed. Hemolysis is not a feature of FH. Sitosterolemia is distinguished from FH by episodes of hemolysis. It is a rare autosomal recessive disorder that causes a marked increase in the dietary absorption of plant sterols. Hemolysis is due to incorporation of plant sterols into the red blood cell membrane. Sitosterolemia is confirmed by demonstrating an increase in the plasma levels of sitosterol using gas chromatography. CT scanning of the liver does not sufficiently differentiate between the hyperlipoproteinemias. Many of the primary lipoproteinemias, including sitosterolemia, are inherited in an autosomal recessive pattern, and thus a pedigree analysis would not be likely to isolate the disorder.

98. The answer is C.

(Chap. 31) This patient has nephrotic syndrome, which is likely a result of multiple myeloma. The hyperlipidemia of nephrotic syndrome appears to be due to a combination of increased hepatic production and decreased clearance of very low-density lipoproteins, with increased LDL production. It is usually mixed but can manifest as hypercholesterolemia or hypertriglyceridemia. Effective treatment of the underlying renal disease normalizes the lipid profile. Of the choices presented, HMG-CoA reductase inhibitors would be the most effective to reduce this patient’s LDL. Dietary management is an important component of lifestyle modification but seldom results in a greater than 10% fall in LDL. Niacin and fibrates would be indicated if the triglycerides were higher, but the LDL is the more important lipid abnormality to address at this time. Lipid apheresis is reserved for patients who cannot tolerate the lipid-lowering drugs or who have a genetic lipid disorder refractory to medication. Cholesterol ester transfer protein inhibitors have been shown to raise highdensity lipoprotein levels, and their role in the treatment of lipoproteinemias is still under investigation.

99. The answer is E.

(Chap. 37) The patient presents with prehypertension, as evidenced by systolic blood pressure of 120–139 mmHg or diastolic blood pressure of 80–89 mmHg. Although at this blood pressure medication therapy is not indicated, the MRFIT trial clearly showed a graded influence of both systolic and diastolic blood pressure on cardiovascular mortality including down to within normal range at 120 mmHg systolic. Thus, lifestyle modification is in order for the patient described here. Alcohol consumption is recommended to be two or fewer drinks per day for men and one drink or less per day for women. NaCl consumption of less than 6 g per day has been shown to reduce blood pressure in patients with established hypertension and in certain ethnic groups. To reduce blood pressure, regular moderate to intense aerobic activity for 30 minutes 6–7 days per week is recommended. Finally, a weight loss of 9.2 kg has been shown to drop blood on average 6/3 mmHg.

100. The answer is C.

(Chap. 37) Several factors have been shown to confer an increased risk of complications from hypertension. In the patient described here there is only one: ongoing tobacco use. Epidemiologic factors that have poorer prognosis include African-American race, male sex, and onset of hypertension in youth. In addition, comorbid factors that independently increase the risk of atherosclerosis worsen the prognosis in patients with hypertension. These factors include hypercholesterolemia, obesity, diabetes mellitus, and tobacco use. Physical and laboratory examination showing evidence of end-organ damage also may portend a poorer prognosis. This includes evidence of retinal damage or hypertensive heart disease with cardiac enlargement or congestive heart failure. Furthermore, electrocardiographic evidence of ischemia or left ventricular strain but not left ventricular hypertrophy alone may predict worse outcomes. A family history of hypertensive complications does not worsen the prognosis if diastolic blood pressure is maintained at less than 110 mmHg.

101 and 102. The answers are D and E, respectively.

(Chap. 37) This patient presents at a young age with hypertension that is difficult to control, raising the question of secondary causes of hypertension. The most likely diagnosis in this patient is primary hyperaldosteronism, also known as Conn’s syndrome. The patient has no physical features that suggest congenital adrenal hyperplasia or Cushing’s syndrome. In addition, there is no glucose intolerance, as is commonly seen in Cushing’s syndrome. The lack of episodic symptoms and the labile hypertension make pheochromocytoma unlikely. The findings of hypokalemia and metabolic alkalosis in the presence of difficult to control hypertension yield the likely diagnosis of Conn’s syndrome. Diagnosis of the disease can be difficult, but the preferred test is the plasma aldosterone/renin ratio. This test should be performed at 8 A.M., and a ratio above 30 to 50 is diagnostic of primary hyperaldosteronism. Caution should be taken in interpreting this test while the patient is on ACE inhibitor therapy, as ACE inhibitors can falsely elevate plasma renin activity. However, a plasma renin level that is undetectable or an elevated aldosterone/renin ratio in the presence of ACE inhibitor therapy is highly suggestive of primary hyperaldosteronism. Selective adrenal vein renin sampling may be performed after the diagnosis to help determine if the process is unilateral or bilateral. Although fibromuscular dysplasia is a common secondary cause of hypertension in young females, the presence of hypokalemia and metabolic alkalosis should suggest Conn’s syndrome. Thus, magnetic resonance imaging of the renal arteries is unnecessary in this case. Measurement of 24-hour urine collection for potassium wasting and aldosterone secretion can be useful in the diagnosis of Conn’s syndrome. The measurement of metanephrines or cortisol is not indicated.

103. The answer is B.

(Chap. 38) For all patients with aortic dissection or hematoma, appropriate management includes reduction of shear stress with beta blockade and management of systemic hypertension to reduce tension on the dissection. However, emergent or urgent surgical therapy is indicated to patients with ascending aortic dissection and intramural hematomas (type A), and for complicated type B dissections (distal aorta). Complications that would warrant surgical intervention include propagation despite medical therapy, compromise of major branches, impending rupture, or continued pain. Thus, patient B has a distal dissection without evidence of complications and is the best candidate for medical therapy.

104. The answer is B.

(Chap. 38) Abdominal aortic aneurysms (AAAs) affect 1–2% of men older than age 50. Most AAAs are asymptomatic and are found incidentally on physical examination. The predisposing factors for AAA are the same as those for other cardiovascular disease, with over 90% being associated with atherosclerotic disease. Most AAAs are located infrarenally, and recent data suggest that an uncomplicated infrarenal AAA may be treated with endovascular stenting instead of the usual surgical grafting. Indications for proceeding to surgery include any patient with symptoms or an aneurysm that is growing rapidly. Serial ultrasonography or CT imaging is imperative, and all aneurysms larger than 5.5 cm warrant intervention because of the high mortality associated with repair of ruptured aortic aneurysms. The rupture rate of an AAA is directly related to size, with the 5-year risk of rupture being 1–2% with aneurysms less than 5 cm and 20–40% with aneurysms greater than 5 cm. The mortality of patients undergoing elective repair is 1–2% and is greater than 50% for the emergent treatment of a ruptured AAA. Preoperative cardiac evaluation before elective repair is imperative, as coexisting coronary artery disease is common.

105. The answer is E.

(Chap. 38) Aortitis and ascending aortic aneurysms are commonly caused by cystic medial necrosis and mesoaortitis that result in damage to the elastic fibers of the aortic wall with thinning and weakening. Many infectious, inflammatory, and inherited conditions have been associated with this finding, including syphilis, tuberculosis, mycotic aneurysm, Takayasu’s arteritis, giant cell arteritis, rheumatoid arthritis, and the spondyloarthropathies (ankylosing spondylitis, psoriatic arthritis, Reiter’s syndrome, Behçet’s disease). In addition, it can be seen with the genetic disorders Marfan’s syndrome and Ehlers-Danlos syndrome.

106. The answer is E.

(Chap. 38) Descending aortic aneurysms are most commonly associated with atherosclerosis. The average growth rate is approximately 0.1–0.2 cm yearly. The risk of rupture and subsequent management are related to the size of the aneurysm as well as symptoms related to the aneurysm. However, most thoracic aortic aneurysms are asymptomatic. When symptoms do occur, they are frequently related to mechanical complications of the aneurysm causing compression of adjacent structures. This includes the trachea and esophagus, and symptoms can include cough, chest pain, hoarseness, and dysphagia. The risk of rupture is approximately 2–3% yearly for aneurysms less than 4 cm and increases to 7% per year once the size is greater than 6 cm. Management of descending aortic aneurysms includes blood pressure control. Beta blockers are recommended because they decrease the contractility of the heart and thus decrease aortic wall stress, potentially slowing aneurysmal growth. Individuals with thoracic aortic aneurysms should be monitored with chest imaging at least yearly, or more frequently if new symptoms develop. This can include CT angiography, MRI, or transesophageal echocardiography. Operative repair is indicated if the aneurysm expands by more than 1 cm in a year or reaches a diameter of more than 5.5–6.0 cm. Endovascular stenting for the treatment of thoracic aortic aneurysms is a relatively new procedure with limited long-term results available. The largest study to date included more than 400 patients with a variety of indications for thoracic endovascular stents. In 249 patients, the indication for stent was thoracic aortic aneurysm. This study showed an initial success rate of 87.1%, with a 30-day mortality rate of 10%. However, if the procedure was done emergently, the mortality rate at 30 days was 28%. At 1 year, data were available on only 96 of the original 249 patients with degenerative thoracic aneurysms. In these individuals, 80% continued to have satisfactory outcomes with stenting and 14% showed growth of the aneurysm (LJ Leurs, J Vasc Surg 40:670, 2004). Ongoing studies with long-term follow-up are needed before endovascular stenting can be recommended for the treatment of thoracic aortic aneurysms, although in individuals who are not candidates for surgery, stenting should be considered.

107. The answer is C.

(Chap. 39) The patient presents with classic signs of arterial occlusion with limb pain, physical examination showing pallor, and a pulseless, cold leg. She has no risk factors for central or peripheral atherosclerotic disease; thus angiogram would simply confirm the diagnosis of arterial occlusion, not demonstrate her predisposing condition. In the absence of fever or systemic symptoms, vasculitis and endocarditis are unlikely sources of arterial embolization. She likely had a paradoxical embolism in the context of an atrial septal defect, which was the source of her childhood murmur. Because many of these patients develop pulmonary hypertension with time, she is now at risk for a paradoxical embolism. Although in this context, arterial emboli frequently originate from venous thrombus, the thrombi cannot produce a paradoxical embolism in the absence of right-to-left shunt, such as in a large patent foramen ovale or an atrial septal defect.

108. The answer is E.

(Chap. 42) Doppler echocardiography uses ultrasound reflecting off moving red blood cells to determine flow velocity within a structure, in this case the heart or great vessels. Thus, it is most useful for determining abnormal flow or flow limitation. Specifically, it is useful in defining valvular regurgitation or stenosis, cardiac output when combined with the cross-sectional area, and diastolic filling of the ventricle. Heart failure with preserved ejection fraction is associated with impaired left ventricle relaxation in early diastole and subsequently there is reduced early transmitral flow compared to normal individuals. Although Doppler might be helpful to determine the physiologic consequence of pericardial effusion, i.e., tamponade, two-dimensional (2D) echocardiography is the preferred mode for effusion diagnosis. Similarly, 2D echocardiography is used to calculate ejection fraction and diagnose cardiac masses. Diagnosis of ischemia can be made with the addition of physiologic or pharmacologic stress to echocardiography, but not with Doppler echocardiography.

aQuestions and answers were taken from Wiener C et al (eds): Harrison’s Principles of Internal Medicine Self-Assessment and Board Review, 18th ed. New York: McGraw-Hill, 2012.